Aquifer FM End of Case Questions

Pataasin ang iyong marka sa homework at exams ngayon gamit ang Quizwiz!

A 53-year-old female with a past medical history of diet-controlled hypertension presents to the office with a two-month history of worsening hot flashes. Her menstrual cycles are regular, occurring every 30 to 32 days, but they have gradually lessened in duration, now lasting four to five days instead of the previous six to seven days. Vital signs and physical exam are normal. Which one of the following treatments is most likely to improve the patient's symptoms? A. Black cohosh B. Oral estrogen C. Venlafaxine D. Gabapentin E. Yoga

The answer is B. From FM 17.

A 68-year-old male with a past medical history significant for hypertension and diabetes presents to your office with a three month history of headaches. He describes a pain that has occasionally awoken him from sleep and is often worse in the morning. He denies any weakness or changes in vision. His exam shows 4+ reflexes in the right upper and lower extremities but is otherwise normal. What is the best next step in management for this patient? A. Initiate treatment with sumatriptan B. Neuroimaging C. Reassurance D. Referral to counselor for relaxation exercises E. Sleep study

The answer is B. From FM 18.

An otherwise healthy 57-year-old G2P2 female presents to your office with vaginal bleeding that began one week ago. She began her menses at age 13 and had regular menstrual cycles until the age of 49. She denies any tobacco or alcohol use. Further review of systems is negative. You perform a physical exam, including a vaginal exam, pap smear, and bimanual exam. Which of the following is the most appropriate next step? A. Reassurance B. Trial of oral contraceptives C. Transvaginal ultrasound D. Endometrial ablation E. Referral for hysterectomy

The answer is C. From FM 17.

A 31-year-old female, G1P0 at 26 weeks gestation, presents to the clinic for her gestational diabetes screening. Her pregnancy has been uncomplicated. Vital signs are stable. Which of the following would require a follow up three-hour glucose tolerance test? A. Fasting serum glucose concentration of 91 mg/dL B. Fasting serum glucose concentration of 112 mg/dL, and a one-hour serum glucose concentration of 128 mg/dL C. Fasting serum glucose cterm-32oncentration of 113 mg/dL, and a two-hour serum glucose concentration of 134 mg/dL D. Fasting serum glucose concentration of 130 mg/dL, and a one-hour serum glucose concentration of 158 mg/dL D. Two-hour serum glucose concentration of 146 mg/dL

The answer is D. From FM 14.

Ms. Marcos is a 65-year-old woman with a past medical history of type 2 diabetes, hypertension, and hypercholesterolemia who presents with six months of insomnia despite self-medication with acetaminophen, diphenhydramine, and herbal remedies. Her height is 157 cm (62 in), and weight is 113 kg (250 lbs). When considering a differential diagnosis, which one of the following is a common cause of insomnia in the elderly? A. Asymptomatic coronary artery disease B. Chronic sinusitis C. Hypoparathyroidism D. Pneumonia E. Sleep Apnea

The answer is E. From FM 03.

A 25-year-old female presents with vaginal bleeding and cramping. Her last normal menstrual period was six weeks ago. The patient's vital signs are stable. On speculum exam of the vagina, there is no bleeding from the cervix. A quantitative beta-human chorionic gonadotropin (beta-hCG) level is 1492 mIU/ml. The patient is sent home and told to return to the office in 48 hours. Her beta-hCG on the return visit is 3000 mIU/ml. What is the appropriate next step in the management of this patient? A. Dilation and curettage for non-viable pregnancy B. Methotrexate for ectopic pregnancy C. Progesterone level to confirm pregnancy D. Surgery for ectopic pregnancy E. Ultrasound to confirm intrauterine pregnancy

The answer is E. From FM 12.

A 65-year-old female presents to your office for a routine visit. She is found to have a blood pressure of 146/96 mmHg. You repeat the blood pressure in her other arm and get 148/92 mmHg. Her pulse is 70 and regular. Her last BP reading was one year ago and was 120/76 mmHg. She has no other medical problems. Her BMI is 28. She states that she likes to walk 30 minutes every other day with her husband and has been doing that for years now. What is the most appropriate diagnosis at this time? A. Elevated blood pressure reading B. Secondary hypertension C. Stage 1 hypertension D. Stage 2 hypertension E. White coat hypertension

The answer is A. From FM 08.

A 35-year-old overweight female in good health comes to the clinic for a routine physical. Which of the following screenings are recommended by the U.S. Preventive Services Task Force (USPSTF)? A. Blood pressure screening B. Cholesterol panel C. Complete blood count D. HbA1C E. Thyroid stimulating hormone levels

The answer is A. From FM 09.

A 72-year-old female presents to the clinic one year after having suffered a stroke that left her with some residual left-sided weakness/paralysis. On exam, the left shoulder demonstrates a severe decrease in both active and passive ROM and significant pain. The patient has a history of hypertension, hyperlipidemia, and diabetes. What is the most likely diagnosis? A. Adhesive capsulitis B. Biceps tendonitis C. Osteoarthritis D. Rotator cuff tear E. Subacromial bursitis

The answer is A. From FM 25.

A 55-year-old male with no significant past medical history and generally healthy behaviors presents to clinic for a health care maintenance exam. He says, "I'd like to get tested for all types of cancer." He does not have any family history of cancer. Review of systems is negative for any symptoms of prostate cancer, such as urinary frequency, urgency, retention, hematuria, weight loss, or back pain. He is a lifelong non-smoker, and he doesn't drink alcohol or use recreational drugs. Which of the following screening tests is given either a USPSTF A or B recommendation in favor of its routine use for patients such as this one? A. Colon cancer screening B. ECG screening for coronary artery disease C. Lung cancer screening D. Pancreatic cancer screening E. Prostate specific antigen (PSA) testing

The answer is A. From FM 02.

Ms. Burton is a 45-year-old woman who has never been to a primary care provider. She presents today to establish care and get her health in order. Her concerns today are: fatigue, weakness, numbness, insomnia, feeling sad at times, anhedonia, increased appetite, weight gain, dry skin, and increasing hair loss within the past month. Her vital signs are: Pulse is 78 beats/minute Respiratory rate is 18 breaths/minute Oxygen saturation is 95% Blood pressure is 152/84 mmHg Weight is 147 kg (325 lbs) Body Mass Index is 41 kg/m2 Today, her physical exam is significant for thinning hair, poor dentition, a systolic murmur heard at the left upper sternal border, an obese abdomen, and bilateral knee stiffness and pain on range of motion exam. Remainder of the physical exam is within normal limits. Which laboratory tests or studies can be done to rule out medical causes of insomnia, fatigue, and depression? A. CBC, CMP, and TSH B. Chest-X Ray C. CT head without contrast D. HgbA1c, lipid panel, urine microalbumin E. MRI brain with contrast

The answer is A. From FM 03.

Ms. Rogers is a 75-year-old woman who was found unresponsive in her house by her neighbor who had come over to help clean her house. An empty unlabeled pill container was found next to her on the bathroom floor. She was rushed to the ER, stabilized and is now in ICU on a mechanical ventilator. Which of the following are true regarding suicide in the elderly? A. Approximately 75% of the elderly who commit suicide had visited a primary care physician within the preceding month, but their symptoms went unrecognized. B. Elderly persons attempting suicide are more likely to be married and living with their spouse. C. Elderly persons attempting suicide usually report good sleeping habits. D. Firearms are the most common means of suicide in the elderly. E. Suicidal behaviors increase with age, but rates of completed suicides don't.

The answer is A. From FM 03.

A 19-year-old female with no significant past medical history is the driver in a motor vehicle accident and is brought to the Emergency Department by EMS. She is complaining of severe pain in her right lower extremity that has worsened since the accident. In addition, she has started to notice what she describes as "burning and tingling" in her right foot. On physical exam, her right calf is edematous and tender with tense overlying skin. There is no swelling or tenderness of the right foot or ankle but the right dorsalis pedis and posterior tibial artery pulses are barely palpable. She cannot confirm light touch of the foot and cannot wiggle her toes on command. What is the next best step in the management of this patient? A. Emergent surgical consultation B. Diagnostic imaging of right foot and ankle C. Immobilize leg and ankle with a cast D. Reassurance and icepacks q 2 hours E. Urgent EMG of the right lower extremity

The answer is A. From FM 04.

A 41-year-old male with no significant past medical history is brought to the Emergency Department after falling to the ground in the middle of a pick-up basketball game with friends. He did not lose consciousness nor hit his head when he fell. As he landed on the ball of his foot after having taken a shot, he recalls hearing a popping sound followed by immediate pain in the posterior right ankle. On physical exam, the posterior right ankle is edematous and tender to palpation. He is unable to plantarflex his right foot. What is the most likely diagnosis of his current condition? A. Achilles tendon rupture B. Ankle arthritis C. Ankle ligament sprain D. Ankle tendonitis E. Calcaneal fracture

The answer is A. From FM 04.

A 30-year-old male with PMH significant for one month of progressive hoarseness and fever presents with a painful neck mass. He reports increased sweating, racing heart, diarrhea, and fatigue. His vital signs are: Temperature is 39.5 C (103.1 F) Pulse is 97 beats/minute Respiratory rate is 19 breaths/minute Blood pressure is 130/70 mmH On physical exam, you palpate a hard, fixed, painless nodule on the left side of the thyroid gland. Bloodwork results show: TSH: decreased Free T4: increased Which of the following is the most likely diagnosis for this patient? A. Carcinoma of the thyroid B. Graves disease C. Multinodular goiter D. Pituitary adenoma E. Thyroiditis

The answer is A. From FM 05.

A 62-year-old female presents for follow-up of her hypertension and diabetes. In general, her chronic diseases are well controlled and she has suffered no target organ damage. She has worked hard to begin exercising, and is walking vigorously five times a week. She has also worked hard on dietary changes, and has been following the DASH eating plan very seriously. She quit smoking three months ago. Her blood pressure today is 148/88 mmHg, pulse is 72 beats/minute, respiratory rate is 16 breaths/minute, temperature is 37.1 C (98.7 F), and BMI is 32 kg/m2. She is taking metformin 500 mg twice daily, simvastatin 20 mg daily and hydrochlorothiazide (HCTZ) 25 mg daily, and she is adherent with her daily medications. Her labs today include an A1C of 6.6, an LDL of 88 and a basic metabolic panel within normal limits. Which of the following management steps today do you consider the most appropriate? A. Add amlodipine 5 mg daily B. Change her simvastatin from 20 mg to 40 mg C. Impress upon her the importance of making more lifestyle modifications D. Increase HCTZ to 50 mg daily E. Make no changes as she is at her treatment goals

The answer is A. From FM 08.

You are seeing a 55-year-old male who presents to the family medicine practice with a two-week history of daily episodes of sudden lightheadedness and palpitations. These seem to occur without provocation and last for approximately five minutes each time. He has not passed out during these episodes, but he has felt the need to sit down when they come. Sitting calmly and waiting seems to make them go away. He reports no chest pain, diaphoresis, jaw pain, or arm pain with these episodes. He has a past medical history of gout and hypertension, for which he takes daily allopurinol and losartan. He does not smoke or drink. He is not under any recent stresses at home or work. On exam, he is in no acute distress. His vital signs are completely normal, as is his cardiopulmonary exam. A recent complete blood count and TSH were normal. Which of the following is the most appropriate next step? A. Order a 48-hour Holter monitor B. Order an echocardiogram C. Order an exercise stress test D. Reassure him that his palpitations are likely benign and that he should focus on a healthy lifestyle E. Transfer him to the emergency department for admission to the hospital

The answer is A. From FM 09.

Ms. Vasquez is a 38-year-old female with a past medical history of sarcoidosis and recently completed a six-month steroid taper. She presents to her primary care physician after two weeks of lower lumbar back pain. She does not recall any trauma but began to feel a sharp pain after bending over to pick up laundry. The pain radiates bilaterally into her anterior abdomen. She has found no relief with over the counter NSAIDs. On physical exam, she has point tenderness along her vertebrae in the L1-L2 region. There are no neurologic deficits and reflexes are intact. Which of the following is the most appropriate next step in management? A. Order a plain x-ray B. Order complete blood count (CBC) C. Reassess in four weeks D. Recommend conservative management E. Refer to spine specialist

The answer is A. From FM 10.

A 51-year-old female comes to you with acute pain and swelling of the knee. Joint fluid analysis confirms the diagnosis of acute gout. She has a past medical history of atrial fibrillation, hypothyroidism, hypertension, and prior treatment for H. pylori infection. Her current medications include losartan, warfarin, levothyroxine, and omeprazole. She is allergic to penicillin medications. Recent laboratory studies revealed normal hemoglobin and hematocrit, blood urea nitrogen, and creatinine levels. Which of the following information from her history would dissuade you from initiating NSAID therapy? A. Currently on warfarin B. Her age C. Hypothyroidism D. Penicillin allergy E. Previous H. pylori infection

The answer is A. From FM 11.

A 59-year-old patient comes to the local ER with a swollen, tender knee that started yesterday. He returned home two days ago from a five-day hike on the Appalachian Trail. He reports no recent or previous injury to the knee and any history of previous inflammatory joint disease. Vital signs: temperature is 36.5 C (97.7 F), pulse is 80 beats/minute, blood pressure is 139/75 mmHg, and respiratory rate is 22 breaths/minute. His lower extremities are marked with abrasions in various stages of healing. He holds the knee in full extension. The knee is swollen, reddened, and tender, and it feels warm. Which of the following is the most appropriate next step? A. Knee aspiration B. Magnetic resonance imaging (MRI) of the knee C. Prescription for colchicine D. Prescription for full-dose acetaminophen E. X-ray of the knee, including sunrise and standing films

The answer is A. From FM 11.

You are seeing a 19-year-old female who suffered a knee injury playing soccer one day ago. The injury involved a sudden deceleration in which she planted her right foot in front of her while running, whereupon another athlete fell against her shin. She felt a pop and sudden pain. She had to be helped off the field and her knee swelled immediately. Today, she reports that she has considerable right knee pain with bearing weight and that her knee feels unstable at times. Her past medical history is unremarkable, and she takes no medications. On exam, her vital signs are perfectly normal. You conduct a knee exam. Which of the following exam maneuvers is most likely to be abnormal in this patient? A. Lachman test B. Laxity to valgus stress C. Laxity to varus stress D. McMurray test E. Posterior drawer test

The answer is A. From FM 11.

A 15-year-old female with a history of allergic rhinitis presents to the clinic with a five day history of productive cough and wheezing that is worse at night. She denies any shortness of breath, chest pain, or fever. The patient states that she has had similar symptoms in the past, especially when seasons change. The only reported past medical history is atopic dermatitis which is well-controlled with an over-the-counter steroid cream. What is the best next step? A. Prescribe a short acting beta agonist inhaler with a short course of oral steroids B. Prescribe a long acting beta agonist inhaler C. Prescribe a daily corticosteroid inhaler D. Prescribe a daily corticosteroid and long acting beta agonist inhaler E. Prescribe antibiotic therapy

The answer is A. From FM 13.

Which of the following is a comorbid condition that needs to be controlled in order to improve asthma symptoms? A. High BMI B. Insomnia C. Atopic dermatitis D. Anxiety E. Anemia

The answer is A. From FM 13.

A 24-year-old male presents to clinic with history of upper abdominal pain, nausea and vomiting. He has previously had his gallbladder removed due to symptomatic gallstones. In reviewing his history, you want to screen for alcohol abuse due to the possibility of pancreatitis. Which one of the following is a sign of alcohol use disorder (AUD)? Choose the single best answer. A. Failure to fulfill work, school or social obligations due to the effects of drinking B. Denial of a drinking problem C. A score of 2 on the AUDIT-C test D. Consumption of 10 drinks per week E. Three drinks per social occasion

The answer is A. From FM 15.

A 47-year-old female presents to the clinic complaining of an "itchy patch" on her skin. On further examination, you note a solid, elevated 1.5cm lesion on the extensor surface of the right forearm. How would you best describe the lesion? A. Plaque B. Papule C. Patch D. Nodule E. Macule

The answer is A. From FM 16.

A 35-year-old female with a history of headaches presents to your office with a headache that is not responsive to 400 mg Ibuprofen TID or 500 mg Tylenol TID. Upon gathering further history you find that in the past her headaches were controlled with these medications but recently have been worse with this same treatment. She complains of 4-5 headaches weekly that are often present on awakening. Which of the following is the best treatment for her disorder? A. Discontinue use of Ibuprofen and Tylenol B. Prescribe a calcium-channel blocker for headache prevention C. Prescribe a different NSAID to take for acute headaches D. Prescribe an opioid medication to take for acute headaches E. Prescribe sumatriptan to take as needed

The answer is A. From FM 18.

Mr. Gill is a 27-year-old male who presents to his primary care physician with post-prandial epigastric burning. The burning is episodic, without associated hematemesis, dysphagia, or odynophagia. In the past, he has episodically used an OTC chewable calcium carbonate to provide relief. Over the past month, he has had to increase the frequency of its use to four times daily. His blood pressure is 120/71 mm Hg. His pulse is 75 beats/minute and regular. Physical exam reveals minimal epigastric tenderness on palpation but is otherwise normal. Which of the following is the most appropriate next step in management? Choose the single best answer. A. Eight-week trial of pantoprazole once a day B. H. pylori IgG serologic testing C. Fecal immunochemical testing (FIT) D. Fecal occult blood testing (FOBT) E. Upper endoscopy with biopsy

The answer is A. From FM 19.

Ms. McSally is a generally healthy 27-year-old female graduate student who presented to your office twelve weeks ago with episodic postprandial epigastric burning. This had been bothering her for nearly six months but she had been busy with her thesis and was unable to find the time necessary for an appointment. She reported this year has been particularly stressful, with limited time resulting in increased consumption of coffee and takeout fast food. At that time, she began an eight-week trial of omeprazole. She returns now with no improvement of her symptoms. She discontinued the omeprazole one month ago because she ran out of the medication. She has no additional symptoms and physical exam is unremarkable. Which of the following is the most appropriate next step in her management? Choose the single best answer. A. Administer a urea breath test B. Begin treatment with triple therapy of pantoprazole, clarithromycin, and azithromycin for 14 days C. Continue omeprazole for another four weeks with close follow-up D. Refer her for an upper gastrointestinal endoscopy E. Switch PPI from omeprazole to pantoprazole

The answer is A. From FM 19.

A 23-year-old female comes to the clinic to discuss infertility. She states she was treated for an STI two years ago, and remembers receiving a ceftriaxone shot and taking a course of doxycycline. She admits to being hospitalized last year due to a severe infection with gonorrhea. She states that she and her fiance have attempted to have a baby for the past year, without success. She reports no dysmennorhea and has regular menses, but has pain during intercourse. Her last menstrual period was 4 weeks ago. Urine hCG is negative. Vital signs stable. Physical exam is unremarkable. What is the most likely cause of the patient's inability to conceive? A. Pelvic inflammatory disease B. Uterine leiomyoma C. Endometriosis D. Candidiasis E. Ovarian cyst

The answer is A. From FM 20.

Which of the following therapies are recommended as first-line therapy for secondary prevention of noncardioembolic TIA or noncardioembolic stroke? A. Aspirin B. Warfarin (Coumadin) C. Pravastatin (Pravachol) D. Recombinant tissue plasminogen activator (t-PA) E. Ticlopidine (Ticlid)

The answer is A. From FM 22.

Mr. and Mrs. Thomas are first-time parents who have brought their 5-day-old newborn son into your office for his first visit. Mom's pregnancy, baby birth, and today's physical exam are all normal. Both parents are sharing the duties of feeding, cleaning, holding, and watching their son without any problems. When Mrs. Thomas steps out of the room to use the restroom, Mr. Thomas asks to speak with you privately. He wants to know if you can prescribe some antidepressant medication for his wife because since giving birth she hasn't been the same. She has been having mood swings multiple times a day, he has found her crying in bed at night after putting their son in his crib, and she gets very irritated and yells at her parents any time they try to help with the baby. What would be the most appropriate advice to give Mr. Thomas? A. This is known as the "postpartum blues"; is quite common for new moms; and is very likely to self-resolve. B. This behavior is very common and a mild antidepressant can help most women get through it. C. The behavior is uncommon at five days postpartum and mental health counseling should be considered. D. This behavior sounds dangerous and hospitalization is in order. E. You should challenge Mr. Thomas on whether he is helping enough with child care.

The answer is A. From FM 24.

Ms. Jones is a first time mother who brings in her 5-day-old son Tommy for his first scheduled newborn visit. Upon reviewing Tommy's hospital records you note that mom's pregnancy was uncomplicated and that Tommy was born full term via spontaneous vaginal delivery and his birth weight was average for gestational age at 3000 grams. Per your discussion with mom, she has been breastfeeding successfully with good technique every two to three hours on demand since delivery. Tommy's physical exam is normal and largely unchanged from the hospital; he is at the 70th percentile for height and head circumference, but his weight has decreased to 2,750 grams. Mom is very concerned by this weight loss and wants to know what she should do. You should recommend: A. Continuing breast feeding with same frequency and return to clinic in 5-7 days B. Hospitalizing Tommy until he shows adequate weight gain for his age C. Increasing the frequency of breastfeeding until Tommy starts gaining weight D. Refer Ms. Jones and Tommy to a lactation consultant E. Supplementing breast milk with 1-2 oz of baby formula until Tommy starts gaining weight

The answer is A. From FM 24.

Ms. Kovacs is a first-time mother you are seeing on postpartum day two in the hospital, with her newborn son Christopher. Her pregnancy was uncomplicated and Christopher was born full-term via spontaneous vaginal delivery with no complications and a birth weight of 7 lbs, 0 oz (3,178 grams). Ms. Kovacs is getting ready to be discharged home today but is concerned because the baby now weighs 6.8 lbs (3,087 grams) and she hasn't been able to produce milk every time Christopher wants to feed. Upon receiving your recommendations, she has been attempting to breastfeed with good technique every two to three hours with strong latching and vigorous suckling. She is still concerned her baby isn't getting enough to eat. Which of the following would be the most appropriate advice for mom? A. Continue to attempt breastfeeding B. Request a lactation consultation to ensure the baby gains weight C. Remain in the hospital until Christopher is having full and consistent feeds with breast milk D. Stop attempting to breastfeed and begin formula E. Switch from breast milk to formula for a few days to allow mom's body to produce more milk, and then attempt breastfeeding again

The answer is A. From FM 24.

A 17-year-old male football player presents to the emergency department with acute onset of right-shoulder pain. The pain started approximately two hours ago when he was tackled during a game while carrying the football. Shoulder swelling and pain were present after injury. He is holding ice to a swollen shoulder, and on exam you notice severe bruising, pain on palpation over the lateral edge of the shoulder, and a raised bump over that same area. There is also severe tenderness over the coracoclavicular ligaments. What is the most likely diagnosis? A. AC joint sprain B. Rotator cuff tear C. Rotator cuff tendonitis D. Subacromial bursitis E. Subacromial impingement

The answer is A. From FM 25.

Ms. Wicket is a 30-year-old female establishing with you as a new patient. Her past medical history is significant for hypothyroidism for which she has been without treatment for five years. She currently takes no medication and has no surgical or family history. She is single, with no children, drinks alcohol socially, and does not use tobacco or other recreational drugs. The only records she has are from blood work that she had done last month significant for a very elevated thyroid stimulating hormone (TSH). What constellation of symptoms would you expect to find on your review of systems? A. Dry skin, sensitivity to cold, constipation, and fatigue B. Palpitations, sweating,lightheadedness, and fatigue C. Depression, frequent bowel movements, sweating and increased appetite D. Tremor, hair changes, difficulty sleeping and heavy menses

The answer is A. From FM 26.

A 63-year-old male with a past medical history significant for hypertension, COPD, and long-term tobacco use is accompanied by his wife to a hospital follow-up clinic appointment. She is very concerned about her husband's recent hospitalization for a COPD exacerbation and asks what can be done to improve her husband's health. Which of the following holds the greatest long-term health benefit for this patient? A. Cessation of tobacco products B. Immunization against pneumococcus C. Prednisone taken daily D. Pulmonary rehabilitation program E. Tiotropium (Spiriva) inhaled daily

The answer is A. From FM 28.

Ms. Michaels is an 80-year-old female with a past medical history of shingles. She comes to your office accompanied by her daughter Jennifer who reports that her mother is forgetting things. Jennifer explains that her mother will ask the same question several times throughout the day. Ms. Michaels also gets confused easily and is more passive than usual. Her memory problem was noticed two years ago after she forgot to pay her bills on multiple occasions. Jennifer now pays her mother's bills and cleans and cooks for her. Ms. Michaels' vital signs are temperature of 99.2 Fahrenheit, blood pressure of 118/70 mmHg, heart rate of 80 beats/minute, and respiratory rate of 12 breaths/minute. Her physical exam is significant for bilateral osteoarthritis hand deformities. She does not have a tremor, nor jerky uncontrolled movements. She is not on any chronic medications. She has no smoking history and does not drink alcohol. CT head shows mild atrophy of the hippocampus. Her MMSE is 20. Which of the following is the most likely diagnosis? A. Alzheimer disease B. Huntington disease C. Lewy bodies D. Prion protein E. Vascular disease

The answer is A. From FM 29.

13-year-old female patient comes to your office for a physical. Her mother is concerned because she complains of menstrual cramps during her period each month. You determine that menarche was earlier that year and her periods have been mostly regular since that time. The pain is in her lower abdomen and is relieved with Ibuprofen and a heat pack. She has no other medical problems and her physical exam is normal. What is the most appropriate treatment for this patient? A. Continue ibuprofen and heat packs; return if worsening B. Copper IUD C. Start an SSRI D. Start danazol E. Start oral contraceptive pills (OCPs)

The answer is A. From FM 32.

An otherwise healthy 58-year-old female presents in your office with a cough, sore throat and fevers for the past five days. Which of the following clinical details would lead you to treat with an antibacterial agent? A. Dullness to percussion and crackles on lung exam consistent with community-acquired pneumonia B. Myalgias consistent with influenza C. Non-erythematous tympanic membrane with clear effusion D. Purulent discharge and sinus tenderness consistent with acute sinusitis E. Wheezing and productive cough consistent with acute bronchitis

The answer is A. From FM 33.

A 63-year old cisgendered female comes into your office for her annual preventive exam. She has hypertension and type 2 diabetes. She is not sexually active. Her blood pressure is 125/80 and her physical exam otherwise is within normal limits. You recommend influenza and zoster vaccination. Her last colonoscopy was eight years ago and her last mammogram one year ago; both were normal. She has never had an abnormal Pap test. At the age of 45 she had a total hysterectomy for fibroids. Of the details provided about this patient, which is an appropriate reason to explain why she does not need a Pap test today? A. She experienced menopause more than 10 years ago B. She had a total hysterectomy for fibroids C. She has never had an abnormal Pap test D. She is 63 years old E. She is not sexually active

The answer is B. From FM 01.

A 48-year-old male with a past medical history that includes hypertension, chronic obstructive pulmonary disease (COPD), and hyperlipidemia presents to clinic as a new patient in October for a general physical exam. History reveals that he has been smoking a pack of cigarettes daily since age 20. He drinks two beers daily. He is intermittently nonadherent with his medications. Review of the state immunization database reveals that the only immunization he has received as an adult was a tetanus diphtheria shot administered 12 years ago. Which of the following vaccine combinations would be most appropriate for this patient? A. Influenza, meningococcal, and zoster B. Influenza, pneumococcal, and Tdap C. Influenza, zoster, and Tdap D. Meningococcal, pneumococcal, and Tdap E. Meningococcal, pneumococcal, and zoster

The answer is B. From FM 02.

A 20-year-old healthy female with no significant past history presents to your clinic with dysuria, lower abdominal discomfort, frequency, and urgency for one day. She notes an odor to her urine but denies any change in its appearance. She reports no vaginal discharge or fever. Review of systems is otherwise negative. The patient denies sexual activity and reports normal menses. There are no known allergies. Her examination is remarkable for mild suprapubic tenderness. Urinalysis: negative; urine HCG: negative. Which of the following would be the next best step in her clinical management? A. Await urine culture results before any treatment B. Initiate antibiotic treatment while awaiting urine culture results C. Insist that a pelvic exam and cervical DNA probe be performed today D. Reassure that this is not a urinary tract infection E. Urine DNA probe

The answer is B. From FM 04.

A 52-year-old female with no past medical history presents to your office with amenorrhea. The patient states that her menstrual cycles previously occurred approximately every 28 to 34 days. However, she has not had a menstrual cycle for the last 10 months. She also endorses insomnia and intermittent dysuria. She denies any headaches, abdominal pain, constipation or diarrhea, changes in hair distribution, or easy bruising. She has lost 15 pounds since her last visit eight months ago, which she attributes to improving her diet and beginning regular exercise. Which of the following tests or pairs of tests is used to confirm menopause? A. Thyroid Stimulating Hormone (TSH) and Free T4 B. Luteinizing Hormone (LH) and Follicle Stimulating Hormone (FSH) C. Prothrombin Time (PT) and International Normalized Ratio (INR) D. Anti-Mullerian hormone E. Testosterone and Dehydroepiandrosterone sulfate (DHEA-S)

The answer is B. From FM 17.

A 67-year-old male presents with one month of palpitations that are associated with weakness and a loss of energy. These episodes seem to occur in no particular pattern and can last for hours at a time. He is asymptomatic today. He has not noticed a change in his weight lately. He denies cold or heat intolerance, chest pain, dark black stools, cough, fever. He has noticed some increased swelling in his ankles at the end of the day. His past medical history included hypertension, mild COPD, and stable coronary artery disease. His medications include atorvastatin, aspirin, metoprolol, tiotropium and albuterol. His vital signs are: Temperature is 36.7 C (98.1 F) Pulse is 82 beats/minute Respiratory rate is 16 breaths/minute Blood pressure is 128/67 mmHg His exam is notable for pink conjunctivae, moist oral mucosae, a normal neck exam, clear lungs, a regular cardiac rate and rhythm, and a normal abdominal exam. He has trace pitting edema in his ankles. Which of the following would be the most appropriate initial test to evaluate the most likely diagnosis for this patient? A. CBC B. EKG C. Pulmonary function tests D. Sleep study E. Thyroid stimulating hormone (TSH)

The answer is B. From FM 05.

A 61-year-old female has recently been diagnosed with type 2 diabetes. Her fasting glucose was 240 mg/dL and her A1C was 8.9%. Her BP has been 148/90 and 146/86 at two separate office visits. Her home BP measurements have been in a similar range. Her creatinine is 0.9 and she has no known heart disease. She currently takes losartan 100 mg daily for a diagnosis of hypertension. Which of the following would be the most appropriate step in managing this patient's blood pressure? A. Make no changes to her medications as her blood pressure is at goal. B. Start amlodipine daily. C. Start furosemide daily. D. Start lisinopril daily. E. Start metoprolol daily.

The answer is B. From FM 06.

A 65-year-old truck driver presents to your office complaining of right calf redness and swelling. He reports that he has had these symptoms off and on for some time but noticed that the redness and swelling on the right has worsened over the past few days and he also noticed fevers and chills. His PMH is significant for DM type 2, COPD, and heart failure. He has a 25-pack-year smoking history. His physical exam shows a temperature of 101.1, 2+ pitting edema of his calves bilaterally, and dry, flaking skin over both calves. His right foot shows interdigital maceration and he has redness, warmth, and tenderness just inferior to the knee. When measuring the circumference of his right leg, you note that it is 36 cm compared to the left, which has a circumference of 34 cm. What is the most appropriate next step in this patient's care? A. Check a D-dimer. B. Initiate treatment with antibiotics. C. Initiate treatment with Low molecular weight heparin. D. Order a lower extremity Doppler. E. Recommend compression stockings and elevation of swollen leg.

The answer is B. From FM 07.

A 54-year-old male with a history of chronic gout and GERD presents to your office for his health maintenance exam. Vital signs today are blood pressure 138/88 mmHg, pulse 65 beats/min, respiratory rate 12 breaths/minute, afebrile, BMI 29 kg/m2. He does not smoke cigarettes or use illicit substances, and he does drink one or two glasses of wine most evenings. He currently jogs three times a week for approximately 30 minutes at a time. He and his partner order takeout food for supper twice per week and otherwise cook at home. He does not particularly like vegetables, but he tries to eat a piece of fruit every day. He has no current concerns, review of systems is negative, and his physical exam is unremarkable. You recommend lifestyle changes. Which of the following changes is most likely to improve his cardiovascular risk? A. Alcohol cessation B. DASH eating plan C. Increased exercise D. Increasing dietary potassium E. Supplementation with vitamin D

The answer is B. From FM 08.

A 60-year-old male with a past medical history of chronic gout, depression, and stage 1 hypertension presents to your office for a follow-up visit. He has been attempting to reduce his blood pressure with behavioral changes, but has had difficulty maintaining the changes. Today, his vital signs are blood pressure is 144/90 mmHg, pulse is 78 beats/minute, respiratory rate is 12 breaths/minute, and temperature is 37.1 C (98.7 F). His recent basic metabolic panel was completely normal. As you consider starting a medication for his hypertension, which of the following medications is most likely to cause an adverse event in this patient? A. Amlodipine B. Hydrochlorothiazide C. Lisinopril D. Losartan E. Metoprolol

The answer is B. From FM 08.

A 24-year-old G1P0 patient at 14 weeks presents with vaginal bleeding and abdominal cramping. On examination her vital signs are: blood pressure 120/75 mmHg, pulse 74/minute, temperature 36.9 C (98.4 F) taken orally, respiratory rate 18/minute, and oxygen saturation 99% on room air. On pelvic examination, there is a small amount of blood in the vagina, the cervical os is open, and there is no cervical or adnexal tenderness noted. On pelvic ultrasound, an intrauterine gestational sac with a yolk sac is seen. What is her diagnosis? A. Complete abortion B. Inevitable abortion C. Missed abortion D. Septic abortion E. Threatened abortion

The answer is B. From FM 12.

A 27-year-old G0P0 female presents to your office with abdominal pain, fever and vaginal discharge. The discharge has an abnormal odor. She reports a new, male, sexual partner. On pelvic exam, there is cervical motion tenderness and adnexal pain during the bimanual exam. The strings from her IUD, placed two years ago, are seen coming from the os. A cervical culture is positive for Chlamydia trachomatis. What risk factor for chlamydia infection is present in this patient? A. Age B. New sexual partner C. Nulliparity D. IUD E. Sexual orientation

The answer is B. From FM 12.

A 32-year-old female presents at your office for a preconception health visit. She is a G2P2, both deliveries were vaginal. Her first child was born with a neural tube defect. According to the USPSTF, what dosage of folate should this patient take daily before she gets pregnant? A. 1 mg B. 4 mg C. 8 mg D. 400 mcg E. 800 mcg

The answer is B. From FM 12.

A 33-year-old G0P0 female presents to your clinic for her first prenatal visit. Her home pregnancy test was positive, she has been experiencing mild nausea for two weeks. Her last normal menstrual period was six weeks ago. What test should her initial prenatal visit include? A. Abdominal ultrasound B. Complete blood count C. Complete metabolic panel D. Glucose challenge test E. Quad screen

The answer is B. From FM 12.

A 42-year-old woman presents with nausea, vomiting and RUQ pain radiating to her back for one day. She reports a history of similar episodes but none have ever lasted for this long of a time period. Her vital signs are HR of 108, BP of 145/90, RR of 20, O2 saturation of 98% and temperature of 100.8F. Her labs reveal the following: WBC = 14.0 x 103/mL AST = 55 U/L ALT = 60 U/L Amylase = 70 U/L Lipase = 7 U/L What is the most likely diagnosis? A. Biliary colic B. Acute cholecystitis C. Acute pancreatitis D. Duodenal ulcer E. Hepatitis

The answer is B. From FM 15.

It is November and you are working in a small, rural emergency department serving a community that is currently going through a flu epidemic. Your next patient is a 4-year-old male who was brought in by his mother for a sore throat and fever that started the night prior. She says he has a cough and is reporting headaches as well. Since last night, he has had a decreased appetite and hasn't been his normal active self. She also wants you to know that he is allergic to eggs and latex and uses an inhaler once a month for asthma-like symptoms. On physical exam, he is tired-appearing but is not acutely dyspneic. His temperature is 39.2 C (102.6 F), pulse is 102 beats/minute, respiratory rate is 21 breaths/minute, and blood pressure is 108/62 mmHg. You note an erythematous throat with normal-sized tonsils without exudate, clear rhinorrhea, and normal-appearing tympanic membranes. His neck is supple without tender lymphadenopathy, and there are scattered rhonchi on auscultation. A rapid strep test was performed and is negative. His last well-child check was 14 months ago, and his mother says she knows he is due for another but her schedule has been too busy. What is the next best step in management? A. Administer an influenza vaccine B. Order a rapid influenza test and a throat culture C. Order a rapid influenza test alone D. Prescribe oseltamivir (Tamiflu) E. Prescribe penicillin V

The answer is B. From FM 21.

You are seeing a 6-year-old female in the family medicine office for a well-child exam. She and her parents have no concerns about her health today. She is developing normally and is thriving in first grade. She has no significant medical history, and she is up to date on her immunizations. On exam, she is well-appearing with normal vital signs except for a BMI of 19.2 kg/m2, placing her at the 96th percentile for her age. The remainder of her examination is normal. Which of the following would be the most appropriate next step? A. Advise diet and exercise with a goal of a 5% weight loss B. Encourage healthy behaviors with a goal of maintaining her current weight C. Order a TSH D. Reassure her parents that since the remainder of her exam is normal, her weight is not a concern E. Referral to a multidisciplinary weight-management program

The answer is B. From FM 21.

A 71-year-old woman presents to the emergency department with acute headache and numbness of the left arm for the past six hours. Past history includes hypertension and diabetes. She takes amlodipine, chlorthalidone, metformin, and rosuvastatin, but ran out of all her medications recently. Her vitals are: Heart rate: 80 beats/minute Blood pressure: 205/110 mmHg Respirations: 16 per minute Body Mass Index: 30 kg/m2 An EKG reveals normal sinus rhythm with a rate of 82 and no ischemic changes. Which of the following is the most likely diagnosis A. Transient ischemic attack B. Hypertensive emergency C. Embolic stroke D. Brain tumor E. Hypoglycemia

The answer is B. From FM 22.

A 55-year-old female with exercise-induced asthma, but no other chronic health problems, presents to the office with left shoulder pain. She is a tennis player, left-handed, and notices pain serving the ball. When you suggest that it might be her rotator cuff your preceptor asks, "what muscles make up the rotator cuff?" A. Supraspinatus, infraspinatus, teres major, subscapularis B. Supraspinatus, infraspinatus, teres minor, subscapularis C. Supraspinatus, infraspinatus, teres minor, deltoid D. Supraspinatus, infraspinatus, teres major, teres minor E. Supraspinatus, infraspinatus, teres minor, rhomboid minor

The answer is B. From FM 25.

You are seeing a 52-year-old male for a full physical at the local free clinic for patients who are uninsured. He has no concerns other than intermittent headaches, which are well controlled with ibuprofen. His family history is positive only for dementia in his mother and stroke in his father. He does not smoke tobacco, drink alcohol, or use illicit drugs. His general physical exam is normal aside from his being overweight. You mention that he is due for colon cancer screening, and he responds that his older brother has been pressuring him to get a colonoscopy. Unfortunately, he is unable to afford this test. There is a local program to provide free colonoscopies to patients at high risk for colon cancer, but your patient would not qualify on the basis of his lack of risk factors. What is the most appropriate next step with this patient regarding colon cancer prevention? A. Recommend yearly abdominal x-rays, which are available at the free clinic B. Recommend annual fecal occult blood testing C. Recommend that he start saving up money for a colonoscopy, as that is the screening test with the best evidence D. Recommend yearly sigmoidoscopy E. Recommend that given his lack of symptoms or risk factors for colon cancer, he can wait until he is aged 60 to start colon cancer screening

The answer is B. From FM 26.

You are seeing a 55-year-old female patient for follow-up who you met two weeks ago at her physical exam. Her review of systems and physical exam were only significant for fatigue. Her blood work shows iron deficiency anemia. She is post-menopausal x 2 years with no significant past medical history or family history but a 10 pack-year smoking history. Prior to establishing with you she hadn't seen a doctor since she was 18 years old. Which of the following would be the most appropriate next step in diagnosis? A. Prescribe oral iron supplementation and pursue no further workup if her fatigue improves B. Refer her for a colonoscopy C. Order a hemoglobin electrophoresis D. Order a pelvic ultrasound E. Recommend fecal occult blood testing now and every year moving forwards

The answer is B. From FM 26.

Your 28-year-old male patient who presented with severe testicular pain has an exam consistent with a testicular torsion. Which of the following imaging studies is best for confirmation? A. Angiography B. Color Doppler US C. CT scan without contrast D. MRI E. X-ray

The answer is B. From FM 27.

A 52-year-old female presents for her third visit this year for productive cough. She has a 34-pack year history but has weaned down to only five cigarettes per day since she began to notice a cough. With her smoking history, you are concerned about the possibility of COPD. Which of the following criteria is included in the GOLD classification for diagnosis of COPD? A. Brain natriuretic peptide >500 B. Post-bronchodilator FEV1/FVC ratio of < 70% C. Flattened diaphragm on lateral chest film D. Left ventricular function <40% E. Oxygen saturation level of < 89%

The answer is B. From FM 28.

A 61-year-old male with a history significant for COPD presents to the emergency department for shortness of breath. Upon exam you see a thin male with perspiration on his forehead. He is having a difficult time answering questions because "he just can't catch his breath." You order an arterial blood gas on the patient. The results are pH 7.22 (7.34-7.44) PaCO2 81 mmHg (35-45 mmHg) PaO2 55 mmHg (75-100 mmHg) . What is the next best step in diagnosis or management? A. Administer Rocephin B. Begin noninvasive mechanical ventilation C. Immunize against influenza D. Provide nicotine replacement patches E. Repeat the test in two hours

The answer is B. From FM 28.

Ms. Brady, a 78-year-old female prevents to your office after six months for follow-up. Her interval history is significant for a TIA three months ago. Today her Montreal Cognitive Assessment (MoCA) is 19. You note that six months ago her MoCA was 22, and nine months ago it was 26. Physical exam shows temperature of 98.8 Fahrenheit, blood pressure of 167/95 mmHg, heart rate of 76 beats/minute, respiratory rate of 14 breaths/minute, and BMI of 23. Chest: regular rate and rhythm, no murmurs; lungs: clear to auscultation; neuro: weakness in the right upper extremity; abdomen: soft, non-tender. She takes atorvastatin and aspirin. Lab studies show Hgb A1c: 5, TSH: 3, B12: 500 pg/mL. Which of the following is the most important recommendation to prevent further disability in this patient? A. Order a mammogram B. Start hydrochlorothiazide C. Start metformin D. Start vegetarian diet E. Weight loss

The answer is B. From FM 29.

You are seeing a 92-year-old male in the hospital. He was admitted after a fall in which he broke his hip. It is now post-op day two from surgery to repair the fracture. His children report that he has been very confused this morning, with varying levels of alertness. Yesterday, he had been recovering well and even participated in physical therapy. Today, he does now know where he is and is at times combative with his care team. His medical history is positive only for hypertension. His medications include amlodipine 5 mg daily and morphine 4 mg IV every 4 hours as needed for pain. His vital signs are all normal today, and his general exam is unremarkable except for expected postoperative changes to his hip. He is somnolent during the examination. Today's labs include a normal CBC and basic metabolic panel. A urinalysis reveals no leukocyte esterase and is nitrite negative. What is the most likely cause of his current mental status? A. Acute stroke B. Morphine C. Pneumonia D. Residual effects of the anesthetic medications from his surgery E. Urinary tract infection

The answer is B. From FM 29.

A 21-year-old G1P0 female presents to the clinic as a new patient to establish prenatal care. Which statement represents something that would not be expected to be a benefit of group prenatal care for this patient? A. Decreases the likelihood of preterm delivery B. Increases adherence to techniques for pain management during labor C. Increases physician contact D. Increases support network E. Shared education between patients

The answer is B. From FM 30.

A 32-year-old female at 33 weeks and 5 days gestation (G2P1) presents to the clinic with headache and RUQ abdominal pain. Blood pressure is 172/121 mmHg on examination while seated. No visual changes noted. Edema is present in the hands, bilaterally. Urine dipstick demonstrated 4+ protein. FHT are 117. Which of the following is the most appropriate next step in the management of this patient? A. Daily aspirin B. Expedited delivery of the premature fetus C. Lisinopril D. Strict bed rest until 37 weeks E. Twice-weekly non-stress testing

The answer is B. From FM 30.

You are seeing a 55-year-old female with a past medical history of hypertension, diabetes, and gout. She has no complaints today and is here to manage her chronic conditions. She is taking her medications, which include lisinopril 10 mg daily, metformin 500 mg twice daily, and allopurinol 100 mg daily. She is trying to focus on her diet in an effort to lose weight. She smokes a pack of cigarettes every day, and she is worried that quitting will make her gain weight and worsen her diabetes. Her family history is positive for a recent myocardial infarction in her father at age 78. Her vitals today include a blood pressure of 128/78 mmHg, pulse of 78 beats/minute, respirations of 14/minute and a BMI of 32 kg/m2. Her general exam is unremarkable today. Which of the following is not a risk factor for coronary artery disease in this patient? A. Hypertension B. Her family history C. Smoking D. Diabetes E. Obesity

The answer is B. From FM 31.

You are seeing a 60-year-old male brought into your office because of "dizzy spells where he almost passes out" for a week. Your records indicate a history of back pain and diabetes. Upon further questioning, he says he sometimes feels like he is going to pass out and gets short of breath after walking about half a block. He's never felt this way before. He denies having a spinning sensation when he is dizzy. His back has also been really bothering him for the past several weeks so he has been taking ibuprofen "all the time." His temperature is 98.6 Fahrenheit; blood pressure is 105/65 mmHg; heart rate is 100 beats/minute; and respiratory rate is 18 breaths/minute. On physical exam, you note no nystagmus, but he does have conjunctival pallor. His lungs are clear to auscultation, and his cardiac exam reveals mild tachycardia with a regular rate and no murmurs. His abdomen is mildly tender and non-distended. As you think through your differential diagnosis and tests to order, which of the following best describes the likely etiology of his symptoms? A. Acute labrynthitis B. Anemia C. Aortic stenosis D. Myocardial infarction E. Thyroid storm

The answer is B. From FM 33.

A 25-year-old patient presents to the office for follow-up on anxiety and tobacco dependence. She reports she is doing well on her new medication to help with both her mood and smoking, though she continues to smoke. She recently started her first sexual relationship with a new female partner. She received a tetanus vaccine at the age of 18, and she received her flu vaccine this year. Her blood pressure is 122/70, and her physical exam is within normal limits. You review her recent Pap test, which was negative. Which of the following indicates the vaccines she should receive today? Choose the single best answer. A. HPV vaccine alone B. HPV vaccine and pneumococcal conjugate vaccine (PCV13) C. HPV vaccine and pneumococcal polysaccharide vaccine (PPSV23) D. HPV vaccine and Zoster vaccine E. No vaccines are needed

The answer is C. From FM 01.

A 55-year-old male comes to the clinic for a visit. He has read about the dangers of being overweight and inquires about which category he fits into. He is 5' 10'' (1.78 m) and weighs 220 lbs (100 kg), BMI = 31.6. Which of the following categories most accurately describes the patient based on his BMI? A. Ideal B. Morbidly (very severely) obese C. Obese D. Overweight E. Underweight

The answer is C. From FM 02.

A 55-year-old male with a family history of melanoma presents to the clinic for evaluation of a skin lesion on his back which appeared three months ago. His wife first alerted him to it, hasn't noticed it change and he has not noticed any symptoms associated with it. Physical examination reveals a 7 mm uniformly black macule that is symmetrically round with sharply demarcated borders on his upper back near the right shoulder. Which of the following characteristics would most justify it being biopsied today? A. Borders B. Color C. Diameter D. Location E. Symmetry

The answer is C. From FM 02.

A 22-year-old female with no significant past medical history experienced an inversion-type injury to her right ankle while playing volleyball. The ankle quickly became edematous, but she used ice and was able to bear weight on the foot. When the patient presents at the family medicine ambulatory practice two days following her injury, minimal swelling is noted and motor functions and sensation are preserved. She has tenderness at the anterior lower lateral malleolus but not inferiorly or posteriorly. You diagnose her with an ankle sprain. You recommend continued relative rest and also tell the patient to keep it elevated and ice it several times during the day to help with the pain and swelling. You inform the patient that immobilization and compression is good for the conservative management of her condition. What is the best compression device to use in this situation? A. Compression stockings B. Elastic wrap C. Semi-rigid ankle support D. Solid cast for eight weeks E. Tape

The answer is C. From FM 04.

A 24-year-old previously healthy female has been exhibiting some concerning new symptoms over the past four months. From her history you gather that she's been having increased diarrhea, tremors, palpitations, and fatigue over this time. Your physical exam reveals a fine tremor in the bilateral upper extremities, sweaty palms, and a smooth, enlarged thyroid gland. You determine that testing for thyroid disease is warranted. Of the following lab results, which would be the best support for the diagnosis of Graves disease? A. High TSH, high free T4 B. High TSH, low free T4 C. Low TSH, high free T4; high, diffuse radioactive iodine uptake D. Low TSH, high free T4; high, focal radioactive iodine uptake in the left lobe of the thyroid E. Low TSH, low free T4, reduced radioactive iodine uptake

The answer is C. From FM 05.

A 45-year-old female presents to your office complaining of left calf pain and swelling. She reports first noticing the pain after her yoga class earlier in the week. She denies having any chest pain or shortness of breath. She has no significant PMH. Her physical exam is significant for tenderness to palpation over the left calf. No edema or redness is present. The circumference of her left calf is 30 cm and the circumference of her right calf is 29 cm. What is the most appropriate next step in diagnosis? A. CRP B. CT angiography C. D-dimer D. Lower extremity Doppler E. MRI

The answer is C. From FM 07.

A 52-year-old female with a history of diabetes and rheumatoid arthritis presents for her annual examination. She works in an office 10 hours a day and rarely gets exercise. Her BMI is 23 and her blood pressure is 152/85. Her previous visit two months ago showed blood pressure of 148/82. Her father had a history of diabetes and her maternal grandmother died of rheumatic heart disease at the age of 42. She admits to marijuana drug use in the past and is a nonsmoker. Which of the following is a major risk factor for coronary heart disease (CHD) that this patient has? A. Age B. Family history C. Hypertension D. Obesity E. Rheumatoid arthritis

The answer is C. From FM 09.

Which of the following patients is an appropriate candidate for an exercise stress test? A. A 44-year-old female with a BMI of 40 kg/m2 and history of asthma B. A 48-year-old female with a history of intermittent anginal episodes that have been controlled on medications C. A 52-year-old female with new atypical chest pain with a history of elevated cholesterol, smoking and family history of coronary artery disease D. A 58-year-old male who presents to the emergency room with constant substernal chest pressure, diaphoresis and shortness of breath

The answer is C. From FM 09.

Mr. Roberts is a 78-year-old male with a significant past medical history of chronic kidney disease stage II, coronary artery disease, and hypertension who presents lumbar back pain. He has also been feeling general malaise and chills over the past few days. On review of symptoms he reports having some difficulty urinating with hesitancy and pain on urination. Currently, his chronic conditions are well managed with metoprolol, lisinopril, and aspirin. He has never smoked. Vital signs: temperature is 38 C (100.4 F), blood pressure is 135/75 mmHg, pulse is 76 beats/minute, and respiratory rate is 15 breaths/minute. Given this history, which of the following physical exam maneuvers would be the most helpful in making the diagnosis? A. Abdominal palpation B. Auscultation for an abdominal bruit C. Digital rectal exam D. Pinprick sensation of the legs E. Straight leg test

The answer is C. From FM 10.

A 35-year-old man presents to clinic with acute onset of abdominal pain. On abdominal exam, you flex the patient's right hip to 90 degrees and take his right ankle in your right hand and with your left hand externally then internally rotate his hip by moving the knee back and forth. The patient denies any abdominal pain with this movement. What physical exam sign did you just perform and what disease is it ruling out? A. Psoas sign to rule out appendicitis B. Psoas sign to rule out cholecystitis C. Obturator sign to rule out appendicitis D. Murphy's sign to rule out appendicitis E. Obturator sign to rule out cholecystitis

The answer is C. From FM 15.

A 35-year-old man presents to clinic with acute onset of constant right upper quadrant abdominal pain. Additionally, he complains of ongoing nausea and vomiting. He denies any past abdominal surgeries or chronic diseases and his only medication is a multivitamin. His AUDIT-10 was positive. What lab(s) would you most likely expect to be abnormal to confirm your diagnosis of acute alcoholic hepatitis? A. GFR and INR B. CRP and alkaline phosphatase C. AST, ALT and total bilirubin D. AST, ALT, INR and WBC E. Lipase and amylase

The answer is C. From FM 15.

A 12-year-old male is brought to the clinic with a pruritic, red, scaly rash in the creases of his elbows. He denies new topical exposures and generally feels well. He has a history of seasonal allergies. Which of the following is the most appropriate course of action? A. Biopsy the skin lesion B. Ask the patient to return to the clinic after three days C. Prescribe topical corticosteroids D. Prescribe oral antibiotics E. Prescribe oral corticosteroids

The answer is C. From FM 16.

A 26-year-old male professional football player comes to the clinic with the concern of hair loss. On examination, the scalp is scaly and erythematous, and certain regions are purulent. There are several circular spots where the hair follicles are no longer present. KOH of skin shows hyphae. What is the most appropriate next step in the management of this patient? A. Punch biopsy of lesion B. Topical griseofulvin C. Oral griseofulvin D. Oral prednisone E. Topical prednisone

The answer is C. From FM 16.

Mr. Rodriguez is a 32-year-old male who presents with three months of postprandial epigastric burning. He reports no relief with antacid therapy. He has no allergies and no significant past medical history. He reports no hematemesis, odynophagia, dysphagia, hoarseness, or sore throat. Physical exam is unremarkable other than epigastric tenderness. H. pylori IgG serology is positive. Which of the following treatment regimens will most likely be successful in treating Mr. Rodriquez? Choose the single best answer. A. Omeprazole daily for eight weeks B. Omeprazole twice daily for four weeks C. Omeprazole twice daily, clarithromycin 500 mg twice daily, and amoxicillin 1 g twice daily for 14 days D. Omeprazole daily, clarithromycin 500 mg twice daily, and tetracycline 500 mg three times daily for 14 days E. Omeprazole twice daily, tetracycline 500 mg three times daily, and metronidazole 500 mg twice daily for 10 days

The answer is C. From FM 19.

A 24-year-old female presents to the clinic complaining of a headache and arm pain. She is accompanied by her boyfriend, who insists on staying in the room with her during the visit. When asked to describe when she noticed the pain and how it occurred, the boyfriend states that the patient hit her head on the bathroom cabinet two days ago, and fell on her side afterwards, hitting her arm. The patient nods in agreement, while looking down at the floor. The next best step in obtaining an accurate history and physical exam is to: A. Direct all questions to the boyfriend, since it appears the patient does not want to discuss her situation with you B. Accuse the boyfriend of domestic abuse and call the police to report him C. Ask the boyfriend to leave the room so you can perform the physical exam, and explain that you always do that part of the exam with just the patient, and that he may join you after that part is completed D. Give the patient referrals to neurology and orthopedics to evaluate the patient's headache and arm pain E. Prescribe acetaminophen 650mg po BID to manage the pain, and tell the patient to return in a week if symptoms have not disappeared

The answer is C. From FM 20.

A 32-year-old female comes to the ED complaining of abdominal pain. She states the pain has been bothering her for the last week, and has progressively become worse. She denies nausea, vomiting, any episodes of diarrhea, or chest pain. She states she was diagnosed with high blood pressure six months ago, and is currently taking medication for it. She is sexually active with her boyfriend, and admits to having some pain during sex. She states she does not feel comfortable telling her boyfriend that she does not want to have sex. Her last menstrual period was two weeks ago. She denies a history of sexually transmitted infections, but says that she has not questioned her significant other because "he would be upset." Vital signs reveal a temperature of 98.7, blood pressure of 142/90 mmHg, heart rate of 82, and respirations of 18. The patient is visibly uncomfortable during the physical exam, which shows a few bruises on her abdomen and lower back, which are at different stages of healing. Which of the following is an appropriate technique for screening for domestic violence? A. Request an appointment with the patient's significant other, and ask him about abusing the patient B. Ask if she had ever been sexually abused as a child C. Ask the patient, "All couples disagree at some point in time. What happens when you and your partner argue or disagree?" D. Use family and friends when an interpreter is needed E. Avoid direct questions about guns in the house or about drug/alcohol use by them or their partner

The answer is C. From FM 20.

You are seeing a 68-year-old woman who has presented to the emergency department with left sided facial droop, aphasia, and left sided weakness in her arm and leg. These symptoms began while she was having lunch two hours earlier. On exam, she is hypertensive and afebrile. Her neurologic exam reveals left-sided hemiparesis with expressive aphasia and left-sided hyperreflexia. When evaluating a person with a possible stroke, when might it be acceptable to administer t-PA? A. If onset of neurologic symptoms was < 8 hrs prior to presentation B. If emergent CT head shows a hemorrhagic stroke C. If onset of neurologic symptoms was < 4.5 hours prior to presentation D. If emergent CT head shows multilobar infarction E. If the patient awoke from an 8-hour sleep with stroke symptoms of uncertain duration

The answer is C. From FM 22.

14-year-old Jennifer is brought to clinic by her mother who is concerned about ADHD. She states Jennifer lacks motivation, regularly does not clean her room, and has to be reminded to do chores around the house. She also is concerned Jennifer does not listen to her and avoids doing homework. Which of the following would most strongly support additional evaluation to assess for ADHD? A. A diagnosis of major depression B. A report card with good grades C. A teacher's report of inattentive behavior D. Fidgety behavior in the car when driving long distances E. Symptom onset with entry to high school

The answer is C. From FM 23.

Samantha and Joe bring in their newly adopted 5-year-old son, Matthew, to your office with the chief concern of sore throat, without cough, that began two days ago. He was born in the United States and was neglected by his biological parents. Samantha and Joe believe Matthew lived in an old building and ate mostly non-nutritious meals. They are unsure if his immunizations are up to date. He is behind on routine immunizations. Upon physical exam, he is playful and interactive. His vital signs are: Temperature: 99.0 F; heart rate: 80; respiratory rate: 19, blood pressure: 95/63 mmHg. He has tender anterior cervical lymphadenopathy, tonsillar exudates and palatal petechiae. Which pairing contains the most appropriate next steps in the management of this patient? A. Empiric antibiotic treatment and recommend immunizations today B. Empiric antibiotic treatment and do NOT recommend immunizations today C. Rapid strep test and recommend immunizations today D. Rapid strep test and do NOT recommend immunizations today E. Reassurance and follow-up in one week

The answer is C. From FM 23.

You are seeing a 42-year-old established female patient in the office for her yearly physical. She has a past medical history of hypertension, and she has no significant family history. She is a teacher at a local high school, and she has three school-aged children. She does not drink alcohol or use illicit drugs, but she has smoked a pack and a half of tobacco for the past 20 years. She would like you to order a colonoscopy because a friend of hers was just diagnosed with colon cancer and has passed away from it. What combination of types of cancer screening all receive an A or B recommendation from the USPSTF for this patient? A. Cervical cancer screening, lung cancer screening, and breast cancer screening B. Cervical cancer screening and breast cancer screening C. Cervical cancer screening only D. Cervical cancer screening, breast cancer screening, and colon cancer screening E. Breast cancer screening and lung cancer screening

The answer is C. From FM 26.

ou are seeing a 32-year-old female in the family medicine ambulatory practice who presents with concern of fatigue. She finds herself simply exhausted at various points of her day, particularly when she is sitting and having to focus on something. When she is busy and physically active, she notices the fatigue less. She reports no recent weight change, skin changes, temperature intolerance, tremulousness, or unusual bleeding. She uses a levonorgestrel IUD and does not have menses on a monthly basis. Her past medical history is positive for a history of seasonal allergies that are well-controlled with loratadine. She does not smoke, drink or use illicit substances. She works as an emergency department nurse at a local hospital, alternating between evening and over-night shifts. On exam, she is well-appearing and has normal vital signs and a BMI of 24 kg/m2. She has no conjunctival pallor. Her thyroid is non-palpable. Her cardiopulmonary exam, abdominal exam, and neurologic exam are essentially normal. Her affect displays concern about her fatigue, but shows a normal range and she is able to smile and laugh at times. Of the following, which is most likely to be the cause of her fatigue? A. Side effects of her medication. B. Iron deficiency anemia. C. Sleep dysfunction due to alteration of her circadian rhythm. D. Hypothyroidism E. Chronic Fatigue Syndrome

The answer is C. From FM 26.

Your 28-year-old male patient with testicular pain has been experiencing pain at a level of 8/10 for two hours. Within how many hours should surgery be performed on a patient with a diagnosed testicular torsion? A. 30 minutes B. 1 hour C. 6 hours D. 18 hours E. 24 hours

The answer is C. From FM 27.

A 67-year-old female with an 80 pack year smoking history presents to an urgent care for worsening shortness of breath. She also reports her chronic cough is more frequent and is producing greenish mucous throughout the day. She is using her inhaler every two hours and has been taking an antibiotic left over from a previous sinus infection. On exam there is a whistling noise when she exhales. What other treatment should you consider at this time? A. Digoxin B. Furosemide C. Prednisone D. Propranolol E. Theophylline

The answer is C. From FM 28.

Ms. Martinez, a 74-year-old female, is brought to your clinic by her husband. He is concerned because his wife, who used to take pride in keeping the house in good shape, has stopped cleaning and caring for her appearance. She recently went shopping and left the stove on. She frequently misplaces her car keys and checkbook. Ms. Martinez reports trouble concentrating and does not have much energy. She has a poor appetite and no longer enjoys knitting. Her Montreal Cognitive Assessment (MoCA) is 28/30. Physical exam: General appearance: well-nourished, appears stated age; HEENT: pupils equal, round and reactive to light, moist mucous membranes; chest: normal S1 and S2, no murmurs; lungs: clear to auscultation bilaterally; abdomen: soft, nontender, non-distended; extremities: no cyanosis, no clubbing; neurologic: no focal deficits; psychiatric: flat affect. Labs: TSH: 2; WBC: 6.0; HgbA1c: 4.5. Which of the following is the most likely diagnosis? A. Delirium B. Dementia C. Depression D. Hypoglycemia E. Normal bereavement

The answer is C. From FM 29.

A 24-year-old G1P0 female at 38 weeks gestation presents to ED complaining of strong lower abdominal contractions that are 10 minutes apart for the last hour. Subsequent cervical examination demonstrates that she is 2cm dilated. FHT are 140 and NST is non-reactive with early decelerations. What is the most appropriate management of this patient? A. Augment labor with oxytocin (Pitocin) B. C-section C. Expectant management D. Immediate vacuum delivery E. Rupture membranes to increase labor

The answer is C. From FM 30.

You are seeing a 72-year-old female with a recent diagnosis of diastolic heart failure (HFpEF) with an ejection fraction of 60% on a recent echo. She was given a diuretic in the ER last week and told to follow up with you. She complains of mild new dyspnea on exertion, orthopnea and lower extremity edema. On exam, her vitals include a blood pressure of 142/86 mmHg, pulse of 84 beats/minute, respirations of 16/minute and oxygen saturation of 98% on room air. Pulmonary exam reveals mild bibasilar crackles, cardiac exam reveals a regular rate and rhythm with no murmurs, and her extremities have 1+ edema to the lower shins. Of the following, which would be the most appropriate choice of medication for this patient? A. Amlodipine B. Digoxin C. Metoprolol succinate D. Hydrochlorothiazide E. Spironolactone

The answer is C. From FM 31.

A 23-year-old female patient comes to your office complaining of bothersome symptoms the week before her period each month. She reports that she has significant breast tenderness, is very irritable, and eats significantly more than she does at any other time during the month. Her coworkers notice the difference in her mood, and it is beginning to affect her interactions with them. The symptoms resolve after her period. She has no other medical problems or significant past medical history. Physical exam is normal. What is this patient's most likely diagnosis? A. Menometrorrhagia B. Premenstrual dysphoric disorder C. Premenstrual syndrome D. Primary dysmenorrhea E. Secondary dysmenorrhea

The answer is C. From FM 32.

A 29-year-old trans male not on hormones presents to your office complaining of very heavy periods. These started about six months ago. He reports that his periods were always normal until six months ago, when he started passing a significant number of clots each day. His cycles occur in a regular pattern but usually last more than eight days. He reports having to change a super-absorbent pad every two to three hours. He is sexually oriented toward women but has never been sexually active. After sensitively asking for permission, you perform a pelvic exam that shows the uterus is small, nontender, and has uniform, smooth contour. On physical exam, you note a moderately enlarged thyroid and dry skin. What is his most likely diagnosis? A. Cervical polyp B. Chronic pelvic inflammatory disease C. Menorrhagia D. Metrorrhagia E. Uterine leiomyoma

The answer is C. From FM 32.

You are seeing a 54-year-old female with a past medical history of kidney stones who presents with a chief complaint of "I have a terrible summer cold." She reports three days of low grade fevers (peak of 100.0 Fahrenheit), cough, sore throat, headaches, and nasal congestion. She denies myalgias. Her temperature today is 99.2 Fahrenheit, respirations 14/minute, pulse 78 beats/minute, and blood pressure of 128/74 mmHg. Her head and neck exam reveals normal tympanic membranes, mildly congested nasal turbinates with thin mucous, erythema of the tonsillar pillars and soft palate without tonsillar enlargement or exudate, and mild anterior cervical lymphadenopathy. Her lungs are clear to auscultation. Which of the following options would be the most appropriate therapeutic option for this patient? A. Echinacea supplementation B. Oseltamivir (Tamiflu) 75 mg twice daily for five days C. Pseudoephedrine (Sudafed) as needed for nasal congestion D. Vitamin C supplementation E. Zinc supplementation

The answer is C. From FM 33.

A 64-year-old cisgendered female who is overweight with well-controlled hypertension comes to your office with concerns of a lump in her breast that she noticed while showering. She reports having no pain, tenderness, or skin changes. A pertinent review of systems is negative. Menarche began at the age of 10. Her first child was born when she was 31 and she had her second and last child at the age of 33. She experienced menopause at the age of 44. Her mother died of colon cancer when she was 65 and her father passed away from metastatic prostate cancer at the age of 70. She has no history of tobacco use ever and occasionally drinks a glass of wine with dinner. Her BMI is 34. Which of the information provided thus far puts the patient at decreased risk for breast cancer? A. Age B. Age at first birth C. Age at menarche D. Age at menopause E. Weight

The answer is D. From FM 01.

A 55-year-old male with no significant past medical history presents for a routine physical exam. He last saw a doctor five years ago. Social history is remarkable for a 35-pack-year tobacco history since the age of 20. He indicates that his wife and children have urged him to quit smoking for the last few months. When you ask him if he has considered quitting, he replies, "I just don't see what the big deal is!" Which stage of change best describes this patient at this time? A. Action B. Contemplation C. Maintenance D. Precontemplation E. Preparation

The answer is D. From FM 02.

Mr. Jones is an 82-year-old man who presents to the office for his six-month chronic disease visit. His diabetes and hypertension are controlled on his usual home medications. He reports that his wife died four weeks ago, and he is now experiencing insomnia most days of the week and fatigue and loss of energy nearly every day; reports decreased enjoyment of his activities, such as playing chess with his neighbor; and is also experiencing loss of appetite but no weight loss. He denies any suicidal ideation and has no previous suicide attempts. Mr. Jones says he often hears his wife's voice while going to bed. He says he goes to church to pray. You are trying to determine if your patient's symptoms are normal grief or if you should diagnose and treat him for Major Depressive Disorder (MDD). Which feature of Mr. Jones' case would suggest MDD rather than a normal grief reaction? A. Change in appetite B. Fatigue C. Hearing wife's voice D. Inability to experience any joy E. Insomnia

The answer is D. From FM 03.

A 45-year-old woman presents to the ED with two months of progressive weakness and fatigue and palpitations. She frequently feels lightheaded upon arising and tires with exertion. In the past six months, she has also experienced heavier menses than usual, often with clots. She has gained three pounds since her annual wellness visit a year ago. Her vital signs are: Temperature is 37.3 C (99.1 F) Pulse is 117 beats/minute Respiratory rate is 22 breaths/minute Blood pressure is 92/67 mmHg She appears pale and diaphoretic. Her neck is supple and non-tender. There is no thyroid enlargement or nodule. Lungs are clear to auscultation bilaterally. Cardiac exam reveals elevated heart rate and a diastolic murmur. Her abdominal exam is normal. Which of the following lab values is most likely in the workup of this patient? A. Anti-thyrotropin receptor antibodies (TRAb) B. Elevated free T4 level C. Elevated thyroid stimulating hormone (TSH) level D. Reduced hemoglobin E. Reduced free T4 level

The answer is D. From FM 05.

A 60-year-old female presents to the office complaining of increased frequency of urination and fatigue for the past several months. She reports no fever, dysuria, back pain, diarrhea or abdominal pain. She has noted some weight loss without working on diet or exercise. Her past medical history is significant for hyperlipidemia and hypertension, for which she takes simvastatin and lisinopril. She is a nonsmoker and consumes one to two glasses of wine per week. Her vitals are: Pulse is 70 beats/minute Blood pressure is 130/70 mmHg Body mass index is 30 kg/m2 Physical examination reveals increased pigmentation in her axilla bilaterally. Her labs are as follows: Random plasma blood glucose: 205 mg/dL Creatinine: 0.8 mg/dL TSH: 2.1 U/L. What is the next most appropriate step in establishing a diagnosis of diabetes mellitus in this patient? A. An oral glucose tolerance test B. Fasting blood glucose C. HgbA1C D. The random blood glucose is sufficient E. Urine microalbumin

The answer is D. From FM 06.

A 65-year-old male with type 2 diabetes mellitus and no other chronic health issues presents to the emergency department with altered mental status. The patient experienced no known head trauma. His vitals are: Temperature is 38.1 C (100.6 F) Pulse is 102 beats/minute Respiratory rate is 16 breaths/minute Blood pressure is 90/74 mmHg He responds when you say his name, and he appears well nourished. His mucous membranes appear very dry. Neurological exam reveals no focal deficits. His plasma glucose is found to be 700 mg/dL. Urinalysis reveals no ketone bodies. What is the most likely diagnosis? A. Cardiac arrhythmia B. Cerebrovascular accident C. Diabetic ketoacidosis (DKA) D. Hyperosmolar hyperglycemic state (HHS) E. Thiamine deficiency

The answer is D. From FM 06.

A 55 year-old bus driver presents to your office complaining of left calf redness and swelling. She reports that she developed these symptoms over the last 3 days, but denies any fevers, chills, chest pain or other associated symptoms. Her PMH is significant for obesity, DM type 2 and HTN. She is a nonsmoker. Her physical exam shows a temperature of 98.2, HR 90, lungs clear to auscultation, and her entire left leg is swollen, with 2+ pitting edema, and her left calf measures 39 cm compared to the right which has a circumference of 35 cm. Initial work up with lower extremity doppler reveals a proximal DVT in her left calf, and BMP confirms normal renal function. Patient has full insurance coverage and a stable home situation. What is the best initial treatment for this patient? A. Administration of unfractionated heparin (UH) overlapping with the initiation of warfarin B. Dabigatran C. Low molecular weight heparin (LMWH) D. Rivaroxaban E. Warfarin

The answer is D. From FM 07.

A 65-year-old truck driver presents to your office complaining of right calf pain and swelling. He has recently returned from a four-day cross-country trip after which he had onset of his current symptoms. The patient reports that the week prior to his four-day trip, he was mostly in bed recovering from a bout of the flu. His PMH is significant for DM type 2 and COPD. He has a 25-pack-year smoking history. He denies having any chest pain or SOB. His physical exam shows 2+ pitting edema of his right leg. When measuring the circumference of his right leg, you note that it is 34 cm compared to the left, which has a circumference of 30 cm. What is the most appropriate next step in diagnosis? A. CRP B. CT angiography C. D-dimer D. Lower extremity Doppler E. MRI

The answer is D. From FM 07.

Mr. Giovanni is a 37-year-old male who drives a delivery truck. He presents to your clinic after acute onset of severe lower back pain that began after lifting a large package while at work. When you enter the room, you find him standing, unable to sit comfortably. On physical exam, he has limited lumbar flexion, reduced to 45 degrees, positive straight leg test at 45 degrees on the left, normal gait, but difficulty with heel walk. He has 4/5 strength on the left with ankle plantar flexion. Strength is preserved on the right. Which additional physical exam finding would be consistent with this man's level of disc herniation? A. 2/5 strength on hip flexion B. Decreased range of motion on lumbar extension C. Decreased rectal tone D. Hypoactive ankle tendon reflex E. Positive Stoop test

The answer is D. From FM 10.

Working at your clinic, you receive a call from a patient of yours, Mr. Smith, a 45-year-old male who was seen three days ago complaining of lower back pain. At that time he had no history of trauma, pain that improved while lying down and no neurologic deficits. He works as a truck driver. He was treated conservatively along with pharmacologic intervention with NSAIDs and muscle relaxants. He calls your office now due to only minimal improvement. And although his symptoms have not changed, he is frustrated with the slow progress, needs to get back to work as soon as possible, and is concerned this might be "something serious." Which of the following is the most appropriate next step in management? A. Ask him to double the dosage of his muscle relaxants B. Obtain a plain film x-ray C. Order an MRI D. Reassure him and schedule a follow-up appointment in a few days E. Schedule him for an appointment immediately

The answer is D. From FM 10.

You are seeing a 62-year-old male with a history of osteoarthritis in his knees and well-controlled hypertension and chronic constipation. His arthritic pain has been disabling in recent months, making it very difficult for him to do his work as a plumber. He has tried full dose acetaminophen in combination with diclofenac, but he still reports 8/10 pain and stiffness. He would like to pursue other treatment options. His current medications include chlorthalidone, diclofenac, acetaminophen, and methylcellulose. On exam, he is in no acute distress and his vital signs are normal. His knees show no warmth, erythema, or gross deformity. They are stable to varus and valgus stress. The Lachman test and McMurray test are both normal. There is moderate crepitus bilaterally. Which of the following would be the most appropriate next step in the management of his pain? A. Prescribe a glucosamine sulfate and chondroitin sulfate combination pill B. Prescribe amitriptyline nightly C. Prescribe oxycodone after reviewing a pain management agreement and performing a urine drug screen D. Refer to physical therapy for strength and mobility training E. Schedule for bilateral intra-articular hyaluronic acid (viscosupplementation) injections

The answer is D. From FM 11.

A 28-year-old male comes to the Emergency Department with shortness of breath, cough and wheezing for the last 4 hours. He states that he was diagnosed with asthma recently, and is currently using inhaled corticosteroid with a long acting B2-agonist daily to control his symptoms. His RR is 34/min; Temp: 98.8 degrees F; O2sat: 88%; BP: 130/85 mmHg. What treatment should be given to this patient first? A. Albuterol breathing treatment using a nebulizer B. Intravenous corticosteroids C. Ipratropium breathing treatment using a nebulizer D. Oxygen E. Intravenous third generation cephalosporin

The answer is D. From FM 13.

A 40-year-old male presents to the clinic with dry cough and wheezing for the past two days. He states that his symptoms began two days ago with a headache for which he took aspirin. He denies fever, but does report some continued shortness of breath. He is a smoker but only smokes 1-2 cigarettes a day for about six months. Physical exam is negative except for bilateral wheezing and erythema on the face. What is the most likely diagnosis? A. COPD B. Foreign body aspiration C. Pneumonia D. Asthma E. Pulmonary embolism

The answer is D. From FM 13.

An 18-year-old non-smoking male comes to the clinic with cough, chest tightness, and difficulty breathing. His past medical history is positive only for allergic rhinitis as well as an undiagnosed chronic cough primarily at night since he was 4 years old. He has no past surgical history and no recent travel. On physical exam, you notice the patient appears in mild distress, has hunched shoulders, is using accessory muscles during respiration, and is only able to talk in two to three word sentences. His vitals are: Pulse 125/min, O2 sat 88%, BP 100/70 mmHg. On auscultation, you hear wheezing on inspiration and expiration throughout both lungs. You also notice a prolonged expiratory phase. What is the most likely diagnosis? A. Cystic Fibrosis B. CHF exacerbation C. Foreign body aspiration D. Severe asthma exacerbation E. Pulmonary embolism

The answer is D. From FM 13.

A 24-year-old female presents to the clinic complaining of nausea and headache for the last week. She denies any fever, changes in bowel movements, or sinus symptoms. She is sexually active with one partner, and admits to not being consistent with her birth control pills. She does not remember the date of her last menstrual period, and reports a history of irregular menstrual cycles. She has not taken a home pregnancy test. Her vital signs reveal a blood pressure of 124/76 mmHg, a pulse of 78/min, respirations of 20/min, temperature of 98.1o F (taken orally), and an oxygen saturation of 98% on room air. The first most appropriate step in management is to: A. Admit the patient to the hospital for monitoring. B. Ask the patient to return to the clinic after she has taken a home pregnancy test, and prescribe Zofran 8mg to control her nausea. C. Obtain a serial serum hCG. D. Obtain a urine hCG. E. Perform a pelvic exam to test for sexually transmitted infections.

The answer is D. From FM 14.

A 27-year old female, G2P1, returns to the clinic for her second prenatal visit. Her labs reveal that her blood type is A Rh-. She states she has done research online, and is concerned that this pregnancy will result in her baby dying if it has a different blood type than her own. To reassure the patient, you explain that her team of health care providers will: A. Perform a cesarean section to prevent fetal demise B. Treat the baby with Rhogam within the first 72 hours after delivery to prevent hemolytic anemia of the newborn C. Treat the mother with penicillin during labor to prevent transmission of gram-positive bacteria D. Treat the mother with Rhogam when she is at 28-weeks gestation to prevent development of antibodies against Rh+ antigens, and if it is determined the neonate is Rh+, the mother will receive a second dose of Rhogam postpartum. E. Treat the mother with Rhogam when she is at 28-weeks gestation to prevent thalassemia in the newborn, and if it is determined the neonate is Rh+ the mother will receive a second dose of Rhogam postpartum

The answer is D. From FM 14.

A 36-year-old female, G3P2 at 21 weeks gestation, returns to clinic for an ultrasound. She currently smokes a half pack of cigarettes per day. Her last two children were delivered by cesarean section. She reports no vaginal bleeding, no urinary symptoms, and no fluid leaking. She states her baby moves "all the time." Her vitals reveal a blood pressure of 130/74 mmHg, a temperature of 98.3o F, a pulse of 82, a respiration rate of 18, and a pulse oximetry of 98%. The ultrasound reveals no fetal abnormalities, but the location of the placenta is partially covering the cervical os. The diagnosis and treatment plan is: A. Placenta previa with admittance to the hospital for fetal and maternal monitoring B. Placenta previa with immediate cesarean section C. Placenta previa with reassurance that no additional monitoring is needed D. Placenta previa with subsequent ultrasound surveillance to monitor for any progression or resolution E. Uncomplicated pregnancy

The answer is D. From FM 14.

A 57-year-old female comes in, concerned about a 1.5 cm dark multicolored mole lateral to her left eye that has been increasing in size over the past six months. A punch biopsy shows pathology indicative of squamous cell carcinoma. What is the most appropriate intervention? A. Three-month follow-up visits for the first year; then every six months B. Avoid artificial sources of UV light, such as indoor tanning C. Cryotherapy extending 4 mm beyond the lesion margins D. Mohs surgery E. Excisional biopsy extending 5 mm beyond lesional border

The answer is D. From FM 16.

A 64-year-old male comes in for a routine physical examination. He notes that over the past few months he has had to get up to urinate in the middle of the night. Benign prostatic hypertrophy (BPH) is on your differential. What other symptom is consistent with BPH? A. Cloudy penile discharge B. Erectile dysfunction C. Hematuria D. Urinary urgency E. Dysuria

The answer is D. From FM 16.

A 56-year-old female with a past medical history significant for hypothyroidism and recurrent urinary tract infections presents to the office to discuss dyspareunia and hot flashes that began approximately one year ago. Her last menstrual cycle was 14 months ago, and her family history is significant for breast cancer in her sister, diagnosed at the age of 47. She has become increasingly bothered by the frequency of her hot flashes and has stopped attending social events with her friends due to these symptoms. Which one of the following is true regarding the risks and benefits of hormone-replacement therapy (HRT)? A. Use of combined estrogen and progesterone therapy decreases the risk of breast cancer. B. Use of unopposed estrogen in patients with an intact uterus decreases the risk of endometrial cancer. C. Use of hormone therapy for less than five years will not affect a patient's risk of coronary artery disease. D. Use of hormone therapy decreases the risk of osteoporotic fractures. E. Both systemic and topical estrogens are equally effective for the treatment of vasomotor symptoms.

The answer is D. From FM 17.

You are seeing a 28-year-old female who presents with gradually worsening headaches. She has had headaches off and on since she was a teen, similar to her mother. She typically awakens with them, and they are associated with nausea and vomiting which can last for much of the day. The pain is throbbing and usually unilateral behind the right eye. Resting in a dark room often helps, and ibuprofen can help as well if she is able to take it at the onset of the headache. Recently, the headaches have become more common, happening two to three times per week. These have been interrupting her job as a paralegal, and she is worried she will be fired. She has tried yoga and mindfulness-based stress reduction, but these have not helped. She does not smoke, drink alcohol, or use illicit drugs. She recently stopped her oral contraceptive pills in an effort to get pregnant. Her vital signs and physical exam are perfectly normal. Of the following, which would be the most appropriate next step in managing this patient? A. Advise her to simply continue the ibuprofen as it has been helpful B. Discontinue the ibuprofen and prescribe acetaminophen instead C. Order a CT scan of her head D. Prescribe amitriptyline nightly E. Prescribe sumatriptan as needed

The answer is D. From FM 18.

You are seeing a 33-year-old male who presents with several weeks of gradually-worsening headaches that he describes as severe and upsetting. They tend to occur three to four times per week and get worse as the day goes on. He describes them as feeling "like a vice squeezing my whole head." He has tried ibuprofen for them, and it has helped some. Unfortunately he never has ibuprofen with him at work where the headaches are more common, because his new boss does not allow medications to be kept at work without a doctor's note. He denies nausea, vomiting, lacrimosis, and photophobia. He is not awakened from sleep by the headaches. He appears mildly distressed, and his vital signs are normal. Examination of his head and neck, fundic discs, and cranial nerves is broadly normal. What is the most appropriate step in management of his headaches? A. Instruct him to discontinue ibuprofen, as he is having rebound headaches. B. Order a CT scan of his head to rule out intracranial pathology. C. Prescribe propranolol to be taken as migraine prophylaxis. D. Prescribe ibuprofen to be taken as needed and provide a note for work. E. Prescribe sumatriptan as needed and provide a note for work.

The answer is D. From FM 18.

You are seeing a 56-year-old male who presents with daily headaches for the past two weeks. He describes them as an intense pressure behind both eyes that is throbbing in nature. He reports that lights and loud noises bother him some, but he has been able to tolerate his usual activities as a short-haul truck driver. The headaches tend to last all day. He has not had nausea, visual changes, fever, or chills. These symptoms have occurred in the context of him quitting tobacco, which he has found difficult. He read online about cluster headaches, and he thinks that is what he is experiencing. He would like you to prescribe oxygen treatment to help with the headaches. Which of the following characteristics is most typical of cluster headaches? A. Patients with them often complain of a pulsating quality of pain. B. The pain is typically periorbital and bilateral. C. They are associated with photophobia and phonophobia. D. They are associated with rhinorrhea, lacrimation, miosis, and ptosis. E. They typically last less than a week.

The answer is D. From FM 18.

Ms. Jones, a 35-year-old female with a significant past medical history of SLE who had been on NSAID therapy for the previous three months, presented four weeks ago with heartburn. At that time, she reported episodic, mealtime epigastric burning radiating to the throat for the past few months. She has had no surgeries. Serologic testing for H. pylori IgG was reported to be positive a few days after her visit and she was begun on triple therapy. She now returns to the office for follow-up. Today she reports no epigastric burning or tenderness. Physical exam is not significant. Which of the following is an accepted indication for performing repeat testing at this visit for H. pylori eradication? Choose the single best answer. A. Age below 40 years B. Documentation is required for all patients with confirmed H. pylori infection C. Positive serologic test prior to therapy D. Restarting of chronic NSAID therapy for SLE E. Treatment with triple rather than quadruple therapy

The answer is D. From FM 19.

Ms. Smith is a 38-year-old female with a significant past medical history of hypercholesterolemia who presents to her primary care physician after four weeks of episodic epigastric discomfort. She reports a recurrent ache-like sensation in the epigastric region occurring about 10 minutes after eating and lasting for several hours. The pain does not appear to be associated with any particular food. She reports no odynophagia or dysphagia. Ms Smith is a smoker, with a 30-pack-year history. You are concerned about peptic ulcer disease. Which additional history finding would increase your level of suspicion? Choose the single best answer. A. Report of hoarseness B. Epigastric discomfort associated with recumbency or bending C. Bouts of recurrent laryngitis D. Recurrent symptoms despite consistent use of antacids E. Sour or bitter taste in mouth

The answer is D. From FM 19.

A 22-year-old female comes to the clinic complaining of frequent urination. She states she has noticed some increased frequency for the past week. She denies itching or pain in the vaginal area, but reports a burning sensation during urination that began a week ago. When asked if she has noticed blood in her urine, she admits to not paying attention to its color. She states that she has some abdominal pain in her pelvic area. She admits to being inconsistent with her birth control pills, and states she is sexually active with both men and women. She does not recall the date of her last menstrual period. Vital signs show a blood pressure of 126/78 mmHg, temperature of 98.2, respirations of 18, heart rate of 82, and oxygen saturation of 97%. Physical exam reveals suprapubic tenderness. She exhibits no costovertebral angle tenderness. Urine hCG test is negative. What is the most likely diagnosis? A. Pyelonephritis B. Urosepsis C. Pregnancy D. Uncomplicated urinary tract infection E. Vaginitis

The answer is D. From FM 20.

A 34-year-old male comes to the clinic complaining of recurrent abdominal pain. He says the pain has been bothering him for the past eight months. He reports episodes of diarrhea and constipation, with more episodes of constipation. He states he has noticed an increase in flatulence. He denies any nausea or vomiting. He has noticed mucus in his stools, but no blood. He states that he cannot recall if anything aggravates the pain, but admits to being under more stress than usual, due to his mother-in-law moving in with him and his wife. Vital signs show a blood pressure of 124/76 mmHg, pulse of 74, respirations of 16, a temperature of 97.9, and oxygen saturation of 98% on room air. Physical exam is unremarkable. The most appropriate initial step in management is: A. Scheduling the patient for a colonoscopy to look for colon cancer B. Discussing the patient's diet, and educating him about avoiding dairy products C. Performing a CBC, TSH, complete metabolic panel, and stool studies D. Offer behavioral therapies and exercise to help with symptoms E. Scheduling the patient for a CT scan of the abdomen to rule out small bowel obstruction

The answer is D. From FM 20.

A 13-year-old female comes to your clinic stating she has been having fever and chills for three days, and aching muscles for the last two days. She states she has also had a mild cough, but is not having any difficulty with breathing. She is up to date on vaccines and her only other medical history is having her tonsils and adenoids removed last year. On physical exam, you find her temperature is 39.2 C (102.6 F), pulse is 96 beats/minute, and her blood pressure is 108/62 mmHg. She has clear rhinorrhea and her oropharynx is mildly erythematous. The rest of her physical exam is normal, and a rapid strep test in the office is negative. What is the next best step in management? A. Albuterol B. Amantadine C. Aspirin D. Ibuprofen E. Zanamivir

The answer is D. From FM 21.

A 57-year-old man presents for follow-up after suffering an ischemic stroke with residual weakness on the left. His past medical history includes type II diabetes. He smokes 1/2 pack per day of tobacco and he does not drink alcohol. He reports no medication allergies. He has at times found the costs of his medications and test strips to be a barrier to adherence. What pairing of an anti-platelet agent and its rational is the best choice for this patient? A. Aspirin 81 mg; more effective at preventing subsequent strokes than higher doses B. Aspirin 325 mg; more effective at preventing subsequent strokes than lower doses C. Aspirin 325 mg; fewer bleeding complications D. Aspirin 81 mg; fewer bleeding complications E. Clopidogrel 75 mg; optimal for patients with diabetes

The answer is D. From FM 22.

James is brought in by his mother and father today for a well-child visit. His mother had a normal term pregnancy and uncomplicated spontaneous vaginal delivery. Thus far James has been meeting all his developmental milestones. Today James can feed himself, easily pedal a tricycle, uses three-five word sentences, can be understood by strangers 75% of the time, can copy a square, takes turns in games, and engages in fantasy play with his siblings. At what developmental age is James? A. 1 year B. 18 months C. 2 years D. 3 years E. 6 years

The answer is D. From FM 23.

A 41-year-old male presents to clinic with left shoulder pain after a fall where he caught himself with his left arm. To assess the injury the patient's arms were placed in 30 degrees of horizontal forward flexion and his shoulders were abducted to 90 degrees. The patient attempted to resist downward pressure and was unable to do so. This test assesses the integrity of the: A. Biceps tendon B. Deltoid muscle C. Infraspinatus muscle D. Supraspinatus muscle E. Teres minor muscle

The answer is D. From FM 25.

A 28-year-old, G2P1 female delivers a 6 lb., 7oz. baby boy at 39 weeks gestation. At one minute, the baby has blue extremities and a pink body; his arms and legs are flexed and he is moving them vigorously with prompt response to stimulation; HR is 118 bpm and he is coughing and crying vigorously as well. What would his APGAR score be at one minute? A. 6 B. 7 C. 8 D. 9 E. 10

The answer is D. From FM 30.

A 56-year-old male presents for care at the ED complaining of dry cough for the past three days. He notes that this problem started a few days after his family's annual fish fry and barbecue and has been worsening since. He has no known past medical history but mentions that he has not seen a doctor in years. He notes that the cough is worse at night often waking him from sleep. He is unable to lie flat on his back and has started using three to four pillows to sleep comfortably. He also reports increased swelling in his legs that worsens throughout the day. He denies having any chest pain or palpitations and also does not believe he has had any sick contacts. He does not know his family history since he was adopted as a child. He has not had any fevers, sweats, or chills. On exam, you observe a tachypneic, obese man in mild distress. On chest auscultation, he has an S3, bilateral crackles at the lung bases, and 2+ pitting edema in the lower legs bilaterally. What diagnostic test would you perform first? A. Exercise stress test B. Pharmacologic stress test C. Echocardiogram D. Electrocardiogram E. Cardiac catheterization

The answer is D. From FM 31.

You are seeing a 63-year-old male with hypertension, diabetes, and a history of an NSTEMI two years ago. His most recent echocardiogram reveals mild hypokinesis of the inferior wall of the left ventricle and a LV ejection fraction of 40%. Shortly after his MI, he was treated for symptoms of congestive heart failure, but he has not has any such symptoms since then. His exercise tolerance is excellent. Today his physical exam is completely unremarkable. The correct pairing of NYHA functional class and ACCF/AHA Stage of CHF is which of the following for this man? A. NYHA II / Stage C B. NYHA I / Stage A C. NYHA I / Stage B D. NYHA I / Stage C E. NYHA II / Stage B

The answer is D. From FM 31.

A 13-year-old female patient comes to your office for a physical. Her mother is concerned because she complains of menstrual cramps during her period each month. You determine that menarche was earlier that year and her periods have been mostly regular since that time. The pain is in her lower abdomen and is relieved with Ibuprofen and a heat pack. She has no other medical problems and her physical exam is normal. What best describes this patient's condition? A. Menorrhagia B. Premenstrual dysphoric disorder C. Premenstrual syndrome D. Primary dysmenorrhea E. Secondary dysmenorrhea

The answer is D. From FM 32.

A 23-year-old female patient comes to your office complaining of bothersome symptoms the week before her period each month. She reports that she has significant breast tenderness, is very irritable, and eats significantly more than she does at any other time during the month. Her coworkers notice the difference in her mood, and it is beginning to affect her interactions with them. The symptoms resolve after her period. She has no other medical problems or significant past medical history. She is in a stable relationship with a female partner. Physical exam is normal. What is the most effective treatment for this patient's condition? A. Copper IUD B. Danazol C. Oral contraceptive pills (OCPs) D. SSRI treatment E. Vitamin B6 supplementation

The answer is D. From FM 32.

Mr. Jones is a 67-year-old male brought into your office because he has been having "dizzy spells and room spinning" for the past two days that are intermittent. Your records indicate a history of back pain, diabetes and hypertension. Upon further questioning, Mr. Jones cannot identify when these spells come on and nothing seems to relieve them. His temperature is 98.6 Fahrenheit; blood pressure is 165/95 mmHg; heart rate is 78 beats/minute; and respiratory rate is 18 breaths/minute. On physical exam, you notice a slight nystagmus. You ask him to focus on your nose but the nystagmus continues. What is the most likely cause of his "dizzy spells"? A. Anemia B. Bleeding gastric ulcer C. Hyperthyroidism with thyroid storm D. Stroke E. Vestibular neuritis

The answer is D. From FM 33.

A 34-year-old cisgendered female who has no past medical problems and is not currently taking any medications comes into your office because she noticed a tender lump in her left breast starting approximately one month ago. She is worried because she has a maternal aunt who had breast cancer that was BRCA positive, though her mother is BRCA negative. Her periods have been regular since they started at the age of 13 and occur every 32 days. She is currently menstruating. She has three children, aged 12, 9, and 4. On exam, her BMI is 32, up from 28 three years ago, and her other vital signs are stable. On breast exam, you note a mobile rubbery mass approximately 1 x 1cm that has regular borders and is tender to palpation. You appreciate no axillary adenopathy. The rest of her physical exam is unremarkable. Of the information provided, which of the following puts this patient at increased risk for breast cancer? A. Age B. Age of menarche C. Family history of cancer D. Parity history E. Weight

The answer is E. From FM 01.

A 47-year-old cisgendered female comes into your office for a health care maintenance exam. She has hypertension and type 2 diabetes. She is not sexually active and has not yet experienced menopause. There is no family history of cancer. Her blood pressure is 118/78, her BMI is 34, and the remainder of her physical exam is within normal limits. Her vaccinations are up to date, and she has a Pap test today and will have labs drawn. According to USPSTF, which of the following is the best recommendation to give her concerning mammography? A. Should have started at age 40 and every year thereafter B. Should have started at age 40 and every two years thereafter C. Should have started at age 45 and every year thereafter D. Start at age 50 and every year thereafter E. Start at age 50 and every two years thereafter

The answer is E. From FM 01.

Ms. Anderson is a 60-year-old woman who comes in to clinic as a walk in appointment. She is tearful and is carrying a box of tissues in her hand. She says she doesn't know why but she has been very sad of late. She reports trouble falling asleep and staying asleep. She used to be the head of her Bridge club, but quit two weeks ago and doesn't feel like going out anymore. She also says she has lost interest in walking her dog, and now just allows him to use the doggie door to let himself out. She also says she feels weak and fatigued and no longer has the energy to do her gardening or shopping. She spends most of her day on the sofa crying while watching TV. She also reports a greatly diminished appetite. She denies suicidal or homicidal ideation, but she does have a history of a previous suicide attempt following her divorce seven years ago for which she was hospitalized. A recent CBC, CMP, CXR, TSH, U/A and CT of the head were all within normal limits. How long do the above symptoms need to be present in order to make the diagnosis of Major Depressive Disorder? A. Eight weeks B. Five weeks C. Four weeks D. One week E. Two weeks

The answer is E. From FM 03.

A 21-year-old female with no significant past medical history experienced an inversion-type injury to her right ankle while playing soccer a day prior to presentation to the family medicine ambulatory practice. She remembers immediate pain and swelling but was able to weight bear and limp off the field. She has noticed some significant swelling which is mostly still present. She has been icing the ankle since the injury as her coach recommended. Pain is still present near the lateral malleolus. Physical examination reveals an edematous lateral right ankle with purplish hue and intact bilateral pulses. Sensation of the bilateral lower extremities is intact and symmetric motor function is preserved. Palpation of the posterior edge of the lateral malleolus elicits significant pain from the patient. There is mild tenderness to palpation of the anterior talofibular ligament and the calcaneofibular ligament. The anterior drawer test and squeeze test are both normal. What is the next best step in the management of this patient? A. Emergent surgical consultation B. Immobilize with cast C. Immobilize with a semi-rigid ankle support D. Rest, ice, ibuprofen, compression, and elevation (RICE) E. X-ray imaging of right ankle

The answer is E. From FM 04.

A 53-year-old male has been experiencing three months of weight loss and palpitations. He reports that the palpitations can come at any time, and that his heart rate increases during these episodes. He also reports feeling nervous more recently. He has not been under any unusual stress in the past three months. He smokes cigarettes and drinks alcohol only on rare occasions. He does not use any illicit substances. Which of the following symptoms or findings, if also present, would be most consistent with a diagnosis of hyperthyroidism, as opposed to hypothyroidism? A. Constipation B. Delayed deep tendon reflex relaxation phase C. Dry skin D. Goiter E. Tremor

The answer is E. From FM 05.

A 42-year-old female presents for a visit after recently being diagnosed with type 2 diabetes. She has made a plan to work on diet and exercise. Her A1C is found to be 8.0%. What is the best medicine to start at this time? A. A sulfonylurea B. An SGLT2 inhibitor C. GLP-1 receptor agonist D. Insulin E. Metformin basal

The answer is E. From FM 06.

A 72-year-old female with a 30-year history of type 2 diabetes and hypertension returns to your office for a routine visit. She is taking 20 units of insulin glargine every morning and five units of insulin aspart with meals. She is on atorvastatin 40 mg daily and lisinopril 40 mg daily. She is on no other medications. Her A1C is 6.5% and her BP today is 145/90. She notes blurry vision for the past several months and a few days of dark spots in her vision. She reports no headaches or nausea. What is the most appropriate next step to slow down the progression of diabetic retinopathy? A. Increase her insulin aspart from five units to seven units with meals. B. Increase her insulin glargine to 23 units every morning. C. Perform a fundoscopic examination and make no changes to her regimen today. D. Start her on a baby aspirin. E. Start her on a calcium channel blocker.

The answer is E. From FM 06.

A 56-year-old Spanish-speaking male presents to your office for a six-month follow-up visit for DM type 2 diagnosed at his previous visit with a HbA1C of 7.0%. At that visit, you discussed dietary modification including eliminating carbohydrates like rice from his diet. You also emphasized the importance of exercise and recommended he join a gym. When he returns for follow up his HbA1c is now 7.5% and he says he has been unable to make the changes you suggested due to food insecurity. Who else from the healthcare team could assist you in helping this patient? A. Dietician B. Endocrinologist C. Nurse D. Pharmacist E. Social Worker/Case Worker

The answer is E. From FM 07.

A 68-year-old male was diagnosed with Stage 1 essential hypertension a few months ago and has been working on diet and lifestyle modifications. He has a BMI of 28 and mild knee arthritis but no other medical diagnoses. He has been a patient of yours for several years, and returns today as planned. Today his blood pressure is 156/94 mmHg. The remainder of his cardiovascular exam is within normal limits. After counseling the patient, he agrees to start an antihypertensive medication. His creatinine is 0.9, urinalysis is normal, and electrolytes are within normal limits. Which of the following is the most appropriate medication to begin in this patient? A. Beta-blocker B. Clonidine C. Loop diuretic D. Nitrate E. Thiazide diuretic

The answer is E. From FM 08.

Which of the following symptoms are most likely to be due to acute coronary syndrome? A. 23-year-old male with acute onset of difficulty breathing and hyperresonance upon lung auscultation B. 35-year-old male with chest pain radiating down his left arm after falling off a ladder at work one week ago C. 42-year-old female with a pulsating pain in the center of her chest at night D. 55-year-old female with diffuse central chest pain that is worse when lying down E. 59-year-old female with palpitations that increase with exercise and are associated with nausea and vomiting

The answer is E. From FM 09.

Mr. Brown is a 42-year-old male accountant with a significant past medical history of obesity who presents to his primary care physician after one week of lower back pain. After moving into a new home three days ago, he woke up the next morning with bilateral lower back pain without any radiation. He denies any recent trauma, fever, chills, numbness, tingling, or incontinence. He has not had any urinary frequency or dysuria. He takes no medications and has no significant past medical history. Which additional findings in his history or physical exam would make the diagnosis of lumbosacral sprain/strain more likely? A. Abnormal gait B. Increased pain with coughing C. Loss of ankle jerk D. Point tenderness on spinous processes E. Spasm of paraspinous muscles

The answer is E. From FM 10.

You are seeing one of your regular patients, a 65-year-old female, for a follow-up appointment for intractable knee pain from osteoarthritis. The knee pain has not responded to ibuprofen or acetaminophen. She has a past medical history that also includes obesity, diabetes, hypertension, and depression. Her current medications include aspirin, HCTZ, metformin, and duloxetine. You are considering prescribing tramadol. She has never taken any kind of opioid medication in the past. Which of the following potential problems should you inquire about when adding tramadol? A. Hypercoagulability B. Hypotension C. Rash D. Recent hyperglycemia E. Seizures

The answer is E. From FM 11.

You are seeing a 32-year-old female at 34 weeks gestation for a routine prenatal visit. Her pregnancy has been uncomplicated, and she is up to date on her immunizations and screening. She is feeling well today, reporting regular fetal movement. She denies vaginal bleeding, leaking of fluid, and contractions. She plans to breastfeed after delivery. Her vital signs are all normal today. You engage in a discussion of postpartum contraception options with her. Which of the following would be recommended as a method of contraception? A. Combined oral contraceptive starting immediately postpartum B. Combined oral contraceptive starting six weeks postpartum C. Depo-levonorgestrel injections starting now, and every three months subsequently D. Etonogestrel/ethinyl estradiol vaginal ring (NuvaRing) starting immediately postpartum E. Levonorgestrel intrauterine device (IUD) inserted immediately after delivery of the placenta

The answer is E. From FM 14.

A 58-year-old man presents to clinic with right upper quadrant pain. The patient has a history of hypertension and GERD, but denies any past surgeries. His abdominal exam reveals a positive Murphy's sign. What would be the next best step to confirm your working diagnosis? A. Complete blood count B. Liver tests C. CT abdomen without contrast D. CT abdomen with contrast E. Abdominal ultrasound

The answer is E. From FM 15.

A 61-year-old G4P4 female presents to a local emergency room with vaginal bleeding. The bleeding began two days ago and is described as spotting. She began her menses at age 16 and had regular menstrual cycles until the age of 59. She endorses smoking ½ pack per day for the last 23 years and drinks 1-2 glasses of wine with dinner every evening. She reports no abdominal pain or dysuria. She is 5'7" and weighs 112 lbs. You perform a physical exam, including a vaginal exam, Pap test, and bimanual exam. Which one of the following characteristics of this patient increases her risk of endometrial cancer? A. Smoking B. Multiparity C. Body habitus D. Age of menarche E. Age of menopause

The answer is E. From FM 17.

A 48-year-old female with no smoking history comes to the emergency department with concerns of sore throat for the past two days. She does not have a thermometer at home, but states she has been feeling hot and her children have also been out from school with fever and sore throat. Her children are now staying with their father whom she is separated from, and he notified her yesterday that they were on antibiotics for their symptoms. She was holding off seeing a clinician because she wasn't coughing or having any problems with swallowing until breakfast this morning. You note that she has a fever of 38.6 C (101.5 F) measured by the nurse, and on physical exam you observe an erythematous throat with exudate, and bilateral cervical lymphadenopathy. Her lungs are clear to auscultation. What is the next best step? A. Chest x-ray B. Empiric levofloxacin (Levaquin) therapy C. Empiric oseltamivir (Tamiflu) therapy D. Empiric penicillin V therapy E. Rapid strep test

The answer is E. From FM 21.

A 7-year-old male is brought to your clinic with a fever of 38.9 to 39.4 C (102 to 103 F) for the past three days. He is up to date on all vaccinations and has no significant medical history. His mother notes that he has not had a cough but is eating and drinking less because "it hurts to swallow." On examination of his neck you notice tender cervical lymphadenopathy bilaterally, and auscultation of his back shows clear lung sounds on both sides. His oropharyngeal exam shows erythematous throat, but no tonsillar exudates. What would be the most appropriate next step? A. Chest x-ray B. Empiric levofloxacin (Levaquin) therapy C. Empiric oseltamivir (Tamiflu) therapy D. Empiric penicillin V therapy E. Rapid strep test

The answer is E. From FM 21.

You are seeing a 72-year-old woman in a family medicine office who presents reporting having had a period of facial drooping and left upper extremity weakness that lasted approximately two hours the prior day. She currently has no symptoms. She has a past medical history of hypertension and Type 2 diabetes, for which she takes losartan and metformin daily. Her family history is positive for coronary artery disease in her father. She is a daily smoker, and she does not drink alcohol. On exam, she has normal vital signs including a blood pressure of 122/74 mmHg. Her cardiac exam reveals a regular rate without murmurs. Her neurological exam is completely normal. Which of the following is the most likely pathophysiology for her presenting symptoms? A.Hemorrhage of a cerebral vessel B. Transient hypotension leading to cerebral ischemia C. Hypertensive urgency that has since resolved D. Acute hypoglycemia E. Blood clot in a cerebral vessel

The answer is E. From FM 22.

Tim is a 15-year-old who comes to clinic with his father, who is concerned Tim has Strep throat. Tim has felt tired for the past day, has a sore throat, scratchy eyes, and dry cough. On exam, his temperature is 101.5 F, heart rate 75, respiratory rate 14, and blood pressure 110/65 mmHg. His pupils are equal, round, and reactive and his conjunctiva are clear. He has increased tearing in both eyes. His nares are patent with mild erythema. His oral pharynx is erythematous without tonsillar enlargement or exudate. He does not have tender or swollen anterior cervical lymph nodes. What is the most appropriate course of action? A. Empiric antibiotic therapy for Strep pharyngitis B. Heterophile antibody test for infectious mononucleosis C. Rapid Antigen Detection Test for Strep pharyngitis D. Strep throat culture E. Symptomatic management without further testing

The answer is E. From FM 23.

The Aronsen family bring their 3-week old newborn for an acute visit because his parents express worry that "something is not right." They report vomiting, which seems more than the post-feeding "dribbles" they were used to with their prior children. The baby seems more irritable and less easily consoled. They deny stool changes, fever, rash, sick contacts, or decrease in frequency of feeds. They notice that he has fewer wet diapers overall. You are concerned that, on exam, the infant does not appear as well as one week prior when he underwent his routine 2-week well baby visit. His weight has increased slightly over the past week, but not as much as expected on the growth curve. He appears to be mildly dehydrated and lethargic, but you find no other distinct physical exam signs. What is the most likely diagnosis? A. Colic B. Failure to thrive C. Gastroesophageal reflux D. Intussusception E. Pyloric stenosis

The answer is E. From FM 24.

You are on call over the weekend for the pediatric clinic and a concerned mother calls in. She has three children at home, her youngest being a 5-week-old. She's very busy and about to leave town on Monday to visit relatives out of state. She tells you that for the last week her two oldest children have had a cough, runny nose, and one or two fevers as high as 38.4 C (101.2 F), but these symptoms have improved in both children with Tylenol. Her youngest child is now having the same symptoms and has a temperature of 38.1 C (100.5 F). Because they're about to go out of town, the mother wants to know if you can call in something stronger like an antibiotic so her youngest can feel better sooner. What is the most appropriate step in managing this child? A. Call in a prescription for amoxicillin as this will cover the likely causative organisms for this infection B. Call in a prescription for a cough syrup so both child and mom can get more sleep. C. Continue to use Tylenol as needed for fevers and supportive care D. Educate mom that this is likely a viral infection and it will have to run its course E. Recommend that the infant be evaluated immediately in your office or the nearest emergency department

The answer is E. From FM 24.

A 23-year-old epileptic male presents to the emergency department after a generalized tonic-clonic seizure. You notice that the patient is holding his right shoulder and that his arm is adducted and internally rotated. What is the best imaging modality for this type of injury? A. CT scan B. MRI C. PET scan D. Ultrasound E. X-ray

The answer is E. From FM 25.

A 16-year-old male patient presents with dull, aching pain in his left scrotum. He says that the dull aching pain is more prominent upon standing. Which of the following the most likely diagnosis? A. Epididymitis B. Henoch-Schönlein purpura (HSP) C. Hydrocele D. Testicular torsion E. Varicocele

The answer is E. From FM 27.

A 21-year-old female comes into your family medicine office complaining of an abnormal discharge with a foul odor. She has been sexually active with four to five partners over the past year, and uses condoms inconsistently. Which of the following can be seen on a wet prep? A. Chlamydia B. Gonorrhea C. Herpes D. Syphilis E. Trichomonas

The answer is E. From FM 27.

A 22-year-old male comes to the office for a physical before he graduates from college. He wants to know which cancer is most common in males in his age range. A. Follicular thyroid cancer B. Gastric cancer C. Pancreatic cancer D. Skin cancer E. Testicular cancer

The answer is E. From FM 27.

A 68-year-old male with GOLD Stage 3, Group D, COPD requiring 2L of oxygen at nighttime presents to clinic complaining of increasing lower extremity edema over the past few weeks. He also thinks his nighttime cough might be worse. His physical exam reveals distant breath sound with scattered rhonchi, a normal cardiac exam, and 2+ bilateral pitting edema in his legs up to his mid shins. What is the most likely mechanism of disease underlying his lower extremity edema? A. Decreased blood flow to the lower extremities due to thromboembolism B. Hepatomegaly from infiltration of the liver with granulomas C. Irregular heart rate due to atrial fibrillation D. Overexpansion of lower extremity veins due to incompetent venous valves E. Pulmonary hypertension causing right heart failure

The answer is E. From FM 28.

A 68-year-old male becomes confused and agitated on post-operative day three after open cholecystectomy. He does not recall his surgery or where he is, and he wants to leave the hospital. Physical exam shows temperature of 99.9 Fahrenheit, blood pressure of 143/89 mmHg, heart rate of 90 beats/minute, respiratory rate of 13 breaths/minute, and O2 sat of 98% on room air. He appears agitated and uncomfortable. He has a Foley catheter. His physical exam is unremarkable. Labs: WBC-11,000, Urinalysis: 2+ nitrites and 2+ leukocyte esterase, 10 WBC/hpf. Glucose finger stick: 80, ECG: normal sinus rhythm at 88 beats per minute. Which of the following is the best initial step to help relieve the patient's symptoms? A. Give IV ceftriaxone B. Give oral ceftriaxone C. Order urine culture D. Order urine gram stain E. Remove urine catheter

The answer is E. From FM 29.

56-year old male with a past medical history of hypertension and type 2 diabetes mellitus presents with progressive dyspnea on exertion, orthopnea, paroxysmal nocturnal dyspnea, and dependent edema over the prior 4 days. His social history is notable for a 30 pack-year tobacco history and occasional marijuana use. His family history is positive for bypass surgery in his mother at the age of 62. On exam, his vitals include a blood pressure of 166/86 mmHg, pulse of 98 beats/minute, respirations of 21/minute and a BMI of 32 kg/m^2. He appears mildly dyspneic. His neck reveals an elevated jugular venous pulse, his chest exam reveals bilateral crackles in the lower lung fields, and his cardiovascular exam reveals mild tachycardia, a regular rhythm, and no murmurs. His abdominal exam is unremarkable, and he has 2+ edema in his legs midway to his knees. An EKG reveals sinus tachycardia with a left axis deviation and Q waves in leads II, III, and AVF. A BNP is measured and is found to be 510 pg/ml. Which of the following is the most likely cause of this patient's heart failure? A. Atrial fibrillation B. Non-ischemic cardiomyopathy C. Valvular disease D. Pulmonary embolism E. Ischemic cardiomyopathy

The answer is E. From FM 31.

You are seeing is a 35-year-old female with no past medical history who presents with dizziness for the past week. She says these episodes of dizziness in which the room feels as though it is spinning last for a minute or two at most but she cannot seem to identify what is causing them. She denies any change in diet, headaches or recent illness. Her father passed away from a stroke at the age of 60 but she denies any other significant family medical history. Her vital signs are all normal, as is her head and neck exam. Her lungs are clear to auscultation bilaterally, and her cardiac exam reveals a regular rate and rhythm with no murmurs. You are unable to elicit saccades with a head thrust maneuver. You perform a Dix-Hallpike maneuver, which elicits her symptoms and causes rotary nystagmus when she looks to the right. What is the most appropriate next step? A. Cardiac enzymes B. ECG C. Emergency CT scan D. Emergency MRI scan E. Epley maneuver

The answer is E. From FM 33.


Kaugnay na mga set ng pag-aaral

BUS 1011: Quiz #2 (Chapters 6-11)

View Set

Chapter 5 : Georgia Laws, Rules, and Regulations Pertinent of All Lines of Insurance

View Set

Chem prep and in class problems exam #3

View Set